Вы находитесь на странице: 1из 58

c

Teora elemental de los nmeros para olimpadas matemticas


David A. SANTOS

Revisin del 17 de junio de 2005

ndice general

Prefacio

III

1. Divisibilidad y primos
1.1. Induccin, buen orden, y principio de Dirichlet . . . .
1.2. Divisibilidad y algoritmo de divisin . . . . . . . . . .
1.3. Mximo comn divisor . . . . . . . . . . . . . . . . .
1.4. Primos y factorizacin nica . . . . . . . . . . . . . .
1.5. Algoritmo de Euclides y ecuaciones diofnticas lineales

.
.
.
.
.

.
.
.
.
.

.
.
.
.
.

.
.
.
.
.

.
.
.
.
.

.
.
.
.
.

.
.
.
.
.

.
.
.
.
.

.
.
.
.
.

.
.
.
.
.

.
.
.
.
.

.
.
.
.
.

.
.
.
.
.

.
.
.
.
.

.
.
.
.
.

.
.
.
.
.

.
.
.
.
.

.
.
.
.
.

.
.
.
.
.

1
1
1
3
4
6

2. Funciones aritmticas
8
2.1. Funciones multiplicativas . . . . . . . . . . . . . . . . . . . . . . . . . . . . . . . . . . . 8
2.2. La funcin parte entera . . . . . . . . . . . . . . . . . . . . . . . . . . . . . . . . . . . . 14
3. Congruencias
3.1. Congruencias . . . . . . . . . . . . . . .
3.2. Sistemas residuales completos y reducidos
3.3. Teoremas de Fermat, Wilson y Euler . . .
3.4. Congruencias simultneas . . . . . . . .
3.5. Criterios de divisibilidad . . . . . . . . .

.
.
.
.
.

.
.
.
.
.

.
.
.
.
.

.
.
.
.
.

.
.
.
.
.

.
.
.
.
.

.
.
.
.
.

.
.
.
.
.

.
.
.
.
.

.
.
.
.
.

.
.
.
.
.

.
.
.
.
.

.
.
.
.
.

.
.
.
.
.

.
.
.
.
.

.
.
.
.
.

.
.
.
.
.

.
.
.
.
.

.
.
.
.
.

.
.
.
.
.

.
.
.
.
.

.
.
.
.
.

.
.
.
.
.

.
.
.
.
.

.
.
.
.
.

.
.
.
.
.

18
18
20
22
23
24

A. Ejemplos y problemas
26
A.1. Sobre el primer captulo . . . . . . . . . . . . . . . . . . . . . . . . . . . . . . . . . . . . 26
A.2. Sobre el segundo captulo . . . . . . . . . . . . . . . . . . . . . . . . . . . . . . . . . . . 35
A.3. Sobre el tercer captulo . . . . . . . . . . . . . . . . . . . . . . . . . . . . . . . . . . . . 43
B. Indicaciones y respuestas

48

Prefacio

He aqu mi pequea contribucin a la difusin de matemticas en castellano.

III

Notacin

N
Z
Q
R
P
]a; b[
[a; b]
]a; b]
[a; b[
]a; +[
[a; +[
] ; a[
] ; a]
TxU
x

Los nmeros naturales {0, 1, 2, 3, . . .}.


Los enteros {. . . , 3, 2, 1, 0, 1, 2, 3, . . .}.
Los nmeros racionales (fracciones).
Los nmeros reales.
Los nmeros primos {2, 3, 5, 7, 11, . . .}.
El intervalo {x R : a < x < b}.
El intervalo {x R : a x b}.
El intervalo {x R : a < x b}.
El intervalo {x R : a x < b}.
El intervalo {x R : x > a}.
El intervalo {x R : x a}.
El intervalo {x R : x < a}.
El intervalo {x R : x a}.
El nico entero que satisface x 1 < TxU x
El nico entero que satisface x < x x + 1

Captulo

Divisibilidad y primos
1.1.

Induccin, buen orden, y principio de Dirichlet

1 Definicin Se denotar los enteros mediante el smbolo Z y los nmeros naturales (enteros positivos,

incluyendo al 0) mediante el smbolo N. Un entero natural p > 1 es primo si sus nicos divisores son p
mismo y la unidad 1. Se denotar el conjunto de los primos mediante el smbolo P. Si un entero diferente
de 1 no es primo, entonces se dice que es compuesto. Obsrvese que 1 ni es primo ni compuesto.
Se presumir que el lector est al tanto de los siguientes tres axiomas, que son, en realidad, equivalentes.
2 Axioma (Axioma del buen orden) Todo conjunto no vaco de nmeros naturales contiene un elemento

mnimo.
3 Axioma (Principio (dbil) de la induccin matemtica) Si S es un conjunto de enteros tal que 0

S y si n S = n + 1 S , entonces S = N.

4 Axioma (Principio (fuerte) de la induccin matemtica) Si S es un conjunto de enteros tal que m

S y si {m, m + 1, . . . , n} S = n + 1 S , entonces k m se tiene k S .


El siguiente razonamiento es usado tan a menudo que amerita un nombre.

5 Regla (Principio de las casillas (pichoneras) de Dirichlet) Si n+1 palomas vuelan hacia n pichon-

eras deber existir al menos una casilla que tenga dos o ms palomas.

1.2.

Divisibilidad y algoritmo de divisin





6 Definicin (Divisibilidad) Se dice que un entero d 6= 0 divide a otro a, denotado por d a, si existe un

entero d tal que dd = a. El caso en que d no dividiere a a se denotar por d a.

7 Teorema (Propiedades de la divisibilidad) Sean a, b, c, d, x, y enteros. Entonces

Captulo 1

(d a, d b) = d (ax + by).
(d a, a b) = d b.

(d a, a 6= 0) = d a .



(d a, a d) = d = a.
Demostracin: Se tiene que
hay d , d con dd = a, dd = b. As
xa + yb = xdd + ydd = (xd + yd )d,



lo que implica que d (xa + yb).


hay d , a con dd = a y aa = b, de donde

dd a = aa = b = d b.

a


a
el nmero es un entero no nulo, luego 1 = a d .
d
d

por definicin da 6= 0. Existen d , r ambos diferentes de 0, tales que dd = a y ar = d.


Entonces dd r = ar = d. Luego d r = 1 y como d , r son enteros, se tiene que d = 1,
r = 1. Se colige que d = a.

8 Teorema (Algoritmo de divisin) Sean b Z \ {0} y a Z. Entonces hay enteros nicos q y r tales

que

a = bq + r, 0 r < b .
a
Demostracin: Si b > 0 tmese q = T U y r = a bq. En virtud de la definicin de la
b
funcin mayor entero,
a
q < q + 1,
b



luego bq a < bq + b, de donde 0 r < b = b . Si acaso b < 0 entonces pngase q =


a
T U.
b

Mximo comn divisor

Para demostrar la unicidad, supngase que a = bq + r = bq + r, con 0 r , r < b . Luego



r r = b(q q ), de donde b (r r). Pero r r < b , lo que requiere que r = r, en


virtud del Teorema 7. De aqu tambin se deduce q = q.
9 Definicin En la ecuacin a = bq + r, a es el dividendo, b 6= 0 el divisor, q el cociente y r el residuo.

El algoritmo de divisin crea particiones de los enteros segn el residuo que stos dejen al ser divididos
por un entero no nulo. Por ejemplo, si n = 5 el algoritmo de divisin dice que los enteros se pueden arreglar
en las siguientes cinco columnas:
..
..
..
..
..
.
.
.
.
.
10 9 8 7 6
5 4 3 2 1
0
1
2
3
4
5
6
7
8
9
..
..
..
..
..
.
.
.
.
.
El arreglo aqu mostrado evidencia que los enteros vienen en uno de cinco sabores: aquellos cuyo residuo
es 0 al ser divididos por 5, aquellos cuyo residuo es 1 al ser divididos por 5, etc. Dicho de otra manera,
todo entero es de la forma 5k, 5k + 1, 5k + 2, 5k + 3 5k + 4. Obsrvese adems que se puede decir
que todo entero es de la forma 5k, 5k 1 5k 2. El algoritmo de divisin pues discrimina y crea clases
entre los enteros, llamadas clases de equivalencia, que se denotarn (en el caso cuando el divisor es 5) por
5Z = {. . . , 15, 10, 5, 0, 5, 10, 15, . . .} = 0,
5Z + 1 = {. . . , 14, 9, 4, 1, 6, 11, 16, . . .} = 1,
5Z + 2 = {. . . , 13, 8, 3, 2, 7, 12, 17, . . .} = 2,
5Z + 3 = {. . . , 12, 7, 2, 3, 8, 13, 18, . . .} = 3,
5Z + 4 = {. . . , 11, 6, 1, 4, 9, 14, 19, . . .} = 4,
y se pondr
Z5 = {0, 1, 2, 3, 4}.
Estas clases se estudiarn ms a fondo en la seccin sobre Zn .

1.3.

Mximo comn divisor


2

10 Definicin (Mximo comn divisor) Sea (a, b) Z , (a, b) 6= (0, 0). Si r a y r b entonces r es

un divisor comn de a y b. Si d es un divisor comn de a y b tal que cualquier otro divisor comn de a
y b divide a d, entonces d es el mximo comn divisor de a y b, denotado por MCD (a, b). Obsrvese
que esto requiere que d sea > 0.

11 Definicin Dcese que dos enteros a y b son relativamente primos si MCD (a, b) = 1.

Captulo 1

12 Teorema (Teorema de Bachet-Bzout) Sea (a, b) Z2 , (a, b) 6= (0, 0). Si d = MCD (a, b) en-

tonces hay enteros x, y tales que

ax + by = d.
Demostracin: Considrese el conjunto
S = {n Z : n > 0, n = as + bt, (s, t) Z2 }.
S 6= ya que o bien a S o bien b S . Luego por el buen orden de los enteros,
S tiene un elemento
mnimo estrictamente positivo al que se llamar n0 = ax0 + by0 . Ob

srvese que d n0 en virtud del Teorema 7 ya que d a y d b. Ahora bien, por el algoritmo de
divisin existen enteros q, r con a = qn0 + r, 0 r < n0 . Si r 6= 0 entonces
r = a qn0 = a q(ax0 + by0 ) = a(1 qx0 ) qby0 S
que es menor que n0 , contradiciendo la definicin
de n0 , de donde se concluye que r = 0.

De la misma manera se puede demostrar que n0 b. Luego n0 es un divisor comn de a y b,



por lo tanto divide a d. Ya que d n0 y n0 d se tiene, en virtud del Teorema 7, se tiene que
d = n0 . Como ambos d > 0, n0 > 0 se colige que d = n0 .

1.4.

Primos y factorizacin nica





13 Lema (Lema de Gauss) Si d ab y MCD (d, a) = 1 entonces d b.

Demostracin:

Por el Teorema de Bachet-Bezout (Teorema 12) existen


enteros x, y tales

que ax + dy = 1. Luego bax + bdy = b. Como d ab se tiene que d (bax + bdy) = b.





14 Lema (Lema de Euclides) Si p es primo, y p ab o bien p a o bien p b.

Demostracin:
Si p a entonces MCD (p, a) = 1. Gracias al Lema de Gauss (Lema 13) se

tiene que p b.
15 Teorema (Teorema fundamental de la aritmtica) Todo entero n > 1 puede descomponerse en fac-

tores de la manera

r
1
n = p
1 pr ,

en donde p1 < p2 < < pr son primos. Esta representacin es nica, a la cual se llamar la factorizacin
cannica de n..
Demostracin: Si n > 1 es primo, entonces no hay nada que demostrar. Supngase que n > 1
es el menor entero no primo que no se puede descomponer de la manera dicha. Como n no

Primos y factorizacin nica

es primo, existen n > 1, n > 1 con n = n n . Pero luego n y n son menores que n y por
1
t
s

1
tanto se pueden descomponer como n = q
1 qs y n = q1 qt donde algunos de los
exponentes pueden ser 0. Se sigue que
1 +1 2 +2
n = n n = n = q
q2
,
1

lo que contradice la suposicin de que n no se poda descomponer en primos.


Para demostrar la unicidad de la descomposicin, se argir por induccin. Supngase que
todo entero mayor que 1 y menor que n puede descomponerse en primos de manera nica. Si
1
r
s
1
n = p
1 pr = q1 qs ,

en donde p1 < p2 < < pr y q1 < q2 < < qr . Entonces por el Lema de Euclides (Lema 14)
p1 debe dividir a exactamente una a de las qs y q1 debe dividir a exactamente una de las
ps. Pero esto fuerza p1 = q1 . Luego al dividir por p1 uno y otro lado se obtiene se obtiene
n
1 1 2
1 1
r
= p
p
q2 qs s .
r = p1
1
p1
n
se descompone en primos de manera nica, luego r = s,
p1
pi = qi y 1 1 = 1 1, 2 = 2 , . . . , r = r , de donde se colige que 1 = 1 y por lo
tanto n tambin se descompone en primos de manera nica.

Por la hiptesis de induccin

16 Teorema (Euclides-Infinitud de los primos) El conjunto P de los primos es inagotable. An ms, si

p1 = 2, p2 = 3, . . . y en general pk es el k-simo primo, entonces


pk+1 p1 pk + 1.
Demostracin: Considrese el entero n = p1 pk + 1. Por el Teorema fundamental de la
aritmtica (Teorema 15) o bien n = p1 pk + 1 es primo, o puede descomponerse como un
producto de primos, lo que demuestra la existencia de una lista de primos dividiendo a n, lista
que se llamar P. Ahora bien, al dividir n por los primos de la lista P = {p1 , p2 , . . . , pk },
n deja residuo 1. De aqu se infiere que P P = y luego a cualquier lista finita de primos
puede agregrsele un primo ms, de donde ninguna lista finita de primos es exhaustiva. El
primo mnimo p diviendo a n debe ser mayor que pk y se tiene p n en virtud del Teorema
7. Es claro que pk+1 p.
17 Definicin Un mltiplo comn de dos enteros a, b es un entero no negativo que es divisible por ambos
a y b. Si m es mltiplo comn de a, b y si m divide a todo otro mltiplo comn, entonces m es el mnimo
comn mltiplo, de a, b, al que se denotar por mcm (a, b).

t
1
s
18 Teorema Sean n = q1 1 q
s y n = q1 qt donde algunos de los exponentes pueden ser 0.

Entonces

mn (1 ,1 ) mn (2 ,2 )
q2

MCD n, n = q1

(1.1)

Captulo 1

max (1 ,1 ) max (2 ,2 )
q2
.

mcm n, n = q1
An ms:

(1.2)

nn = MCD n, n mcm n, n .

(1.3)

y si k es un entero positivo entonces

MCD (a, b)k = MCD ak , bk , mcm (a, b)k = mcm ak , bk .

(1.4)

Demostracin: Las dos primeras aserciones son evidentes gracias al Teorema fundamental
de la aritmtica (Teorema 15). Para la tercera asercin basta notar que
mn(, ) + max(, ) = + .
La cuarta se deriva de las primeras dos.

1.5.

Algoritmo de Euclides y ecuaciones diofnticas lineales

Se dar ahora un procedimiento eficaz para hallar soluciones x, y a la ecuacin MCD (a, b) = ax +
by. Este procedimiento se llama el algoritmo de Euclides y opera como sigue. Sean a, b enteros positivos.
Luego de aplicar el algoritmo de divisin repetidamente, se ve que
a
b
r2
..
.

=
=
=
..
.

bq1 + r2 ,
r2 q2 + r3
r3 q3 + r4
..
.

0 < r2 < b,
0 < r3 < r2 ,
0 < r4 < r3 ,
..
.

(1.5)

rn2 = rn1 qn1 + rn 0 < rn < rn1 ,


rn1 = rn qn .
La sucesin de residuos eventualmente llegar a un rn+1 que ser 0 ya que la sucesin b, r2 , r3 , . . . es
una sucesin de enteros montona decreciente la cual no puede tener ms de b trminos estrictamente
positivos.
19 Lema Si b 6= 0, a = qb + r entonces MCD (a, b) = MCD (b, r).



Demostracin: Pngase d = MCD (a, b) , c = MCD (b, r) . Como d a y d b, se sigue que





d (a qb) = r. As pues d es un divisor comn de r y b. Esto implica que d c. Por otra









parte, c r y c b implican que c (qb + r) = a. Luego c es un divisor comn de a y b de


donde c d, lo que acaba la demostracin.
20 Teorema Si rn es el ltimo residuo diferente de 0 encontrado en el algoritmo de Euclides, entonces

rn = MCD (a, b) .

Algoritmo de Euclides y ecuaciones diofnticas lineales

Demostracin: Aplicando el Lema 19 varias veces se ve que


MCD (a, b) = MCD (b, r2 )
= MCD (r2 , r3 )
.
= ..
= MCD (rn1 , rn )
= rn .

De las ecuaciones en 1.5 se ve que


r2 = a bq1
r3 = b r2 q2
r4 = r2 r3 q3
..
.. ..
.
. .
rn = rn2 rn1 qn1 ,

luego es posible expresar MCD (a, b) como una combinacin lineal de a y b trazando estas
igualdades en reversa.
21 Teorema Sean a, b, c son enteros tales que (a, b)|c. Dada una solucin (x0 , y0 ) de la ecuacin

diofntica lineal
ax + by = c,
cualquier otra solucin es de la forma
a
b
x = x 0 + t , y = y0 t ,
d
d
en donde d = (a, b) y t Z.
Demostracin: Es inmediato que si (x0 , y0 ) es una solucin de ax + by = c, entonces
x = x0 + tb/d, y = y0 ta/d es tambin una solucin. Demostrarse ahora que toda otra
solucin es de la forma deseada.
Supngase que (x , y ) satisface ax + by = c. Si adems ax0 + by0 = c entonces
a(x x0 ) = b(y0 y ).
Dividiendo por d = (a, b),
b
a
(x x0 ) = (y0 y ).
d
d
a
Como (a/d, b/d) = 1, se sigue que |(y0 y ), por virtud del Lema de Euclides. As pues
d
a
existe un entero t tal que t = y0 y , sto es, y = y0 ta/d. Se colige entonces que
d
b a
a
(x x0 ) = t ,
d
d d

which is to say x = x0 + tb/d, demostrando el teorema.

Captulo

Funciones aritmticas
2.1.

Funciones multiplicativas

22 Definicin (Funcin aritmtica) Llmase funcin aritmtica a una funcin f : N \ {0} C. f es mul-

tiplicativa si MCD (m, n) = 1 = f(mn) = f(n)f(n). f es completamente multiplicativa si f(mn) =


f(n)f(n) para todos los enteros positivos m y n.

Toda funcin completamente multiplicativa es, a fortiori, multiplicativa. Una funcin


multiplicativa est completamente determinada por sus valores en las potencias de los primos.
1
r
r
1
As, si n = p
1 pr entonces f(n) = f(p1 ) f(pr ). Adems f(1 1) = f(1)f(1) =
(f(1))2 . Luego, o bien f(1) = 1 o bien f(1) = 0. Este ltimo caso hace que la funcin sea
idnticamente 0 por lo cual no se considerar. Por tanto, se tendr por convencin que para
toda funcin multiplicativa f, f(1) = 1.
23 Teorema Sea n > 0 un entero y f, g funciones con

g(n) =

X


d n

Si f es multiplicativa, tambin lo es g.
8

f(d).

Funciones multiplicativas

Demostracin: Tmese adems n > 0 entero, con MCD n, n



= 1. Todo d nn puede

descomponerse en d = d1 d2 , d1 n, d2 n , de donde MCD (d1 , d2 ) = 1. Luego


g(nn ) =



d nn

f(d)

f(d1 d2 )

f(d1 )(d2 )



d1 n,



d2 n



d1 n,
0



d2 n

B
B
B
B
B
B




d1 n

f(d1 )

10
CB
CB
CB
CB
CB
CB
A



d2 n

C
C
C
C
C
C
A

f(d2 ) ,

demostrando la asercin.

24 Definicin (Funcin nmero de divisores) La funcin

d:

N \ {0} N
\ {0}
X
1
n
7


d n

cuenta el nmero de divisores positivos de un entero positivo n.


25 Teorema La funcin

d:

N \ {0} N
\ {0}
X
1
n
7


d n

r
1
es multiplicativa, y si n = p
1 pr es la descomposicin cannica en primos de n, entonces

d(n) = (1 + 1) (r + 1).
Demostracin: Ya que la funcin
d 7 1 es completamente multiplicativa es, a fortiori,
X
1 es multiplicativa en virtud del Teorema 23. Si p es primo,
multiplicativa. Luego d(n) =

d n

10

Captulo 2
p tiene + 1 divisores: 1, p, p2 , . . . , p . As d(p ) = + 1 y
r
1
d(n) = d(p
1 ) d(pr ) = (1 + 1) (r + 1).

26 Definicin (Funcin suma de divisores) La funcin

N \ {0} N
\ {0}
X
n
7
d


d n

suma los divisores positivos de un entero positivo n.


27 Teorema La funcin

N \ {0} N
\ {0}
X
n
7
d


d n

r
1
es multiplicativa y si n = p
1 pr es la descomposicin cannica en primos de n, entonces

(n) =

1 +1
p
1 prr +1 1
1

.
p1 1
pr 1

Demostracin: Ya que la funcin


d 7 d es completamente multiplicativa es, a fortiori,
X
multiplicativa. Luego (n) =
d es multiplicativa en virtud del Teorema 23. Si p es primo,


d n

p tiene por suma de divisores la suma geomtrica


(p ) = 1 + p + p2 + + p =

p+1 1
.
p1

Luego
r
1
(n) = (p
1 ) (pr ) =

r +1 1
p11 +1 1 p
r
.
p1 1
pr 1

r
28 Definicin Dado n = p1 1 p
r , la funcin : N \ {0} N cuenta cuntos primos distintos n tiene

sin contar factores repetidos, esto es (n) = r. La funcin : N\{0} N cuenta cuntos primos distintos
n tiene incluyendo factores repetidos, esto es (n) = 1 + 2 + + r .

Es evidente que tanto como son multiplicativas, an ms, es totalmente multiplicativa. Tambin es evidente que (n) (n) siempre y que (n) = (n) si y slo si
n no tiene ningn factor cuadrado mayor que 1.

Funciones multiplicativas

11

29 Definicin (Funcin de Mbius) Sean p, p1 , . . . , pr primos distintos y

n > 0 entero. La funcin

N \ {0} {1, 0, 1}
n
7 (n)

:
se define como sigue:

si n = 1
1
(n)
(n) =
(1)
si (n) = (n)

0
si (n) > (n)

30 Teorema La funcin de Mbius es multiplicativa.


Demostracin: Sea n = ab, con MCD (a, b) = 1. Si p P y p n entonces p2 debe dividir

o bien a a o bien a b. En todo caso (n) = (a)(b) = 0. Si a = p1 pr , b = q1 qs


donde todas las ps y qs son diferentes, entonces
(n) = (1)r+s = (1)r (1)s = (a)(b),
de donde es multiplicativa.
31 Teorema Si n > 0 es un entero,

X


d n

(d) =

1 si n = 1
0 si n > 1

Demostracin: Pngase

A = d n : (d) = k, p2 d, p P .
!

n
y si d A se tiene (d) = (1)k . La suma requerida es
Entonces #(A) =
k
X


d n

(n)

(d) =

X
k=0

(n)
(1)k .
k

En virtud del Teorema del binomio, (1 1)(n) = 0.


32
XTeorema (Frmula de inversin de Mbius) Sean f y F funciones aritmticas, en donde F(n) =


d n

f(d). Luego

f(n) =

X


d n

(d)F(n/d) =

X


d n

(n/d)F(d).

12

Captulo 2
Recprocamente, si f(n) =

X


d n

(d)F(n/d) para todo entero n > 0, entonces F(n) =

Demostracin: Se tiene
X


d n

(d)F(n/d) =

XXX


d n



d n



ds n

X


s n



n
s d

X


d n

f(d).

f(s)

(d)f(s)

f(s)



d n
s

(d).

Gracias al Teorema 31, la suma interna es diferente de 0 slo cuando ns = 1. As, el nico
trmino que sobrevive en la suma externa el de s = n, que simplifica a f(n).
Recprocamente, si

X


d n

f(d) =

XX

(s)F(d/s)

XX

(d/s)F(s)

XX

(r)F(s)



d n



s d



d n



s d



s n



r n
s



s n

F(s)

X


r n
s

(r).

Utilizando el Teorema 31, la suma interna ser 0 a menos que s = n, en cuyo caso la suma
simplifica a F(n).
33 Definicin (Funcin indicatriz de Euler) La funcin

N \ {0} N \ {0}
n
7 (n)

cuenta el nmero de enteros entre 1 y n que son relativamente primos a n:


(n) = #(k : 1 k n, MCD (k, n) = 1}.

Funciones multiplicativas

13

34 Teorema Sea n 1 entero. Entonces

X


d n

(d) = n.

Demostracin: Para cada divisor d de n, sea Td (n) el conjunto de enteros positivos n


cuyo mximo comn divisor con n es d. Tanto d vara sobre los divisores de n, los conjuntos
Td (n) forman una particin de {1, 2, . . . , n} y por lo tanto
X


d n

Td (n) = n.

Se demostrar de inmediato que Td (n) posee (n/d) elementos. Obsrvese que los elemenn
tos de Td (n) yacen en el conjunto d, 2d, . . . d. Si k Td (n), entonces k = ad, 1 a
d

k n
n
,
n/d y MCD (k, n) = d. Pero entonces MCD
= 1, lo que implica que MCD a,
=
d d
d
1. Se deduce que contar los elementosde Td(n) es equivalente a contar el nmero de enteros
n
= 1. Pero el nmero de estos enteros es precisaa que satisface 1 a n/d, MCD a,
d
mente (n/d). Colegimos que
X
n=
(n/d).

d n

Pero en tanto d vara sobre los divisores de n,

X
n
(n/d) =
vara en reverso. As pues n =
d

X


d n

d n

(d), lo que demuestra el teorema.

35 Corolario Si n 1 es un entero,

(n) = n

X (d)


d n

Demostracin: Esto se deduce de inmediato en combinando el Teorema de inversin de


Mbius (Teorema 32) y el Teorema 34.
36 Corolario La funcin

N \ {0} N \ {0}
n
7 (n)

14

Captulo 2

r
1
es multiplicativa y si n = p
1 pr es la descomposicin cannica en primos de n, entonces

Y
Y
1

1
(p p
)=n
1
.
(n) =
p


p n

p n

Demostracin: Combnese el Corolario 35 con el Teorema 23 para demostrar que es


multiplicativa. Por otra parte, si p es primo, hay p1 enteros positivos p que tienen un
factor comn con p: 1p, 2p, 3p, . . . , p1 p. Por lo tanto (p ) = p p1 de donde
resulta la segunda aseveracin.

2.2.

La funcin parte entera

37 Definicin Sea x R. Llmase suelo de x (o funcin mayor entero de x) al nico entero TxU que

satisface

x 1 < TxU x.
Ntese que cada x R puede escribirse como x = TxU + {x}, en donde 0 {x} < 1 es la parte fraccionaria
de x.
Anlogamente, llmase techo de x (o funcin menor entero de x) al nico entero VxW que satisface
x VxW < x + 1
38 Teorema Sean (, ) R2 , a Z, n N \ {0}. Entonces

1. T + aU = TU + a

TU
2. T U = T
U
n
n
3. TU + TU T + U TU + TU + 1
Demostracin: Se demostrar cada inciso individualmente.
1. Sea m = T + aU. Entonces m + a < m + 1. De aqu m a < m a + 1.
Esto significa que m a = TU, completando la demostracin de este inciso.
2. Pngase /n = T/nU + , 0 < 1. Como nT/nU es un entero, se deduce del
inciso 1 que
TU = TnT/nU + nU = nT/nU + TnU.
Ahora bien, 0 TnU n < n, y por lo tanto, 0 TnU/n < 1. Si = TnU/n,
entonces se obtiene

TU
= T U + , 0 < 1,
n
n
demostrando este inciso.

La funcin parte entera

15

3. De las desigualdades
1 < TU ,

1 < TU

se obtiene + 2 < TU + TU + . Ya que TU + TU es un entero + ,


entonces ser T+U. De aqu se colige que TU+TU T+U. Adems +
es TU + TU + 2, de donde su parte ntegra T + U deber ser TU + TU + 2,
pero como T + U < TU + TU + 2 entonces se tiene T + U TU + TU + 1,
demostrando las desigualdades.

39 Teorema (Formula de De Polignac) La mxima potencia del primo p que divide a n! est dada por

X
n
T k U.
p
k=1

Demostracin: El nmero de factores contribuyendo un factor de p es Tn/pU, el nmero


de factores contribuyendo un segundo factor de p es Tn/p2 U, etc..
40 Teorema Si a, b son relativamente primos, entonces
a1
X
k=1

b1
X ka
(a 1)(b 1)
kb
T U=
U=
.
a
b
2
k=1

Demostracin: Considrese el rectngulo con vrtices en (0, 0), (0, b), (a, 0), (a, b).
Este rectngulo contiene (a 1)(b 1) puntos cuyas coordenadas son ambas enteras. El
xb
rectngulo se divide en dos partes gracias a la recta y =
. Se demostrar que no hay
a
puntos de coordenadas ambas enteras sobre esta recta, a excepcin de los extremos. Si hun b
= . Luego
biese puntos de coordenadas enteras (m, n), 0 < m < a, 0 < n < b, entonces
m a
n/m es una fraccin reducida de la fraccin irreducible b/a, contradiccin. Los puntos
kb
kb
Lk = (k,
), 1 k a 1 estn sobre esta recta. Ahora bien, T U es el nmero de puna
a
kb
), esto es,
tos de coordenadas enteras sobre la recta vertical que va desde (k, 0) hasta (k,
a
a1
X kb
T U, que es el nmero de puntos con coordenadas enteras en la parte inferior del reca
k=1

b1
X

ka
U cuenta el nmero de puntos con coordenadas enteras en
b
k=1
la parte superior del rectngulo. Como hay (a 1)(b 1) puntos de coordenadas enteras
en total, y su nmero es compartido de manera igual por ambas partes, queda demostrado el
teorema.
tngulo. De igual manera,

16

Captulo 2

41 Definicin Llmase espectro de un nmero real a la sucesin infinita

Esp () = {TU, T2U, T3U, . . .}.


Dos sucesiones Esp () y Esp () se dicen complementarias si hacen una particin de los nmeros naturales no nulos, esto es, Esp () Esp () = and Esp () Esp () = N.
Demostrarase ms adelante que las sucesiones
 
Esp 2 = {1, 2, 4, 5, 7, 8, 9, 11, 12, 14, 15, 16, 18, 19, 21, 22, 24, 25, . . .},
and



Esp 2 + 2 = {3, 6, 10, 13, 17, 20, 23, 27, 30, 34, 37, 40, 44, 47, 51, . . .}

son complementarias.
42 Teorema (Teorema de Beatty, 1926) Si > 1 es irracional y

1
1
+ = 1,

entonces las sucesiones
Esp () y Esp ()
son complementarias.
Demostracin: Ya que > 1, > 1, Esp () y Esp () son ambas sucesiones de trminos
distintos y el total de trminos no excediendo N en una y otra sucesin es TN/U + TN/U.
Pero
N/ 1 + N/ 1 < TN/U + TN/U < N/ + N/,
siendo la ltima desigualdad estricta ya que ambos , son irracionales. Como 1/+1/ =
1, se deduce que N 2 < TN/U + TN/U < N. Como la cantidad emparedada es entera,
se deduce que
TN/U + TN/U = N 1.
As, la cantidad total de trminos que no exceden a N en Esp () y Esp () es N 1. Como
esto es cierto para cada N 1, cada intervalo (n, n + 1) contiene exactamente uno de los
trminos de Esp () y Esp (). Se sigue que Esp () Esp () = N, Esp () Esp () = .

Se observa tambin un resultado en la direccin opuesta.


43 Teorema (Teorema de Bang, 1957) Si las sucesiones

Spec() y Spec()
son complementarias, entonces , son nmeros irracionales positivos que satisfacen
1
1
+ = 1.

La funcin parte entera

17

Demostracin: Si ambos , fueren nmeros racionales, entonces eventualmente Spec(),


Spec() contendran los mismos enteros, y por lo tanto no seran disjuntas. As pues, y
son irracionales. Si 0 < 1, dado n entonces existe m para el cual m 1 < n m;
luego n = TmU, lo que implica que Spec() = N, de donde > 1 (y con esto tambin
> 1). Si la interseccin Spec() Spec() fuese finita, entonces
Tn/U + Tn/U
= 1,
n
n
lm

pero entonces (Tn/U + Tn/U)


1/ = 1.

1
1/ + 1/ ya que n , se colige que 1/ +
n

Captulo

Congruencias
3.1.

Congruencias

44 Definicin (Congruencias) Sean n 1, a, b enteros. Dcese que a b mod n (lase a es congruente


a b mdulo n) si a y b dejan el mismo residuo al ser divididos por n, o, equivalentemente, si

n (a b).
45 Teorema (Propiedades de las congruencias) Sean n 1, a, b, c, d enteros. Si a b mod n y

si c d mod n entonces

a + c b + d mod n,
ac bd mod n.

Demostracin: Se tiene n (a b) y n (c d). Luego hay enteros u, v con nu = a b y


nv = c d. As

(a + c) (b + d) = n(u + v)

n ((a + c) (b + d))

a+c b+d

mod n,

y adems
ac bd = a(d + nv)
d(a nu) = n(av + du)

n (ac bd)

ac bd

mod n,

de donde se obtiene el teorema.


46 Corolario Si j 1 es entero y si a b mod n entonces aj bj mod n.

Demostracin: Basta utilizar induccin en el segundo inciso del Teorema 45, poniendo
c = aj1 y d = bj1 .
18

Congruencias

19

47 Corolario Si

p(x) = 0 + 1 x + + n xn
es polinomio cuyos coeficientes son todos enteros y si a b mod n entonces p(a) p(b) mod n.
Demostracin: Utilcese el Corolario 46 para demostrar que i ai i bi mod n y smese
cada trmino.
48 Teorema Sean a, b, n enteros. Si la congruencia ax b

mod n posee al menos una solucin,


entonces tiene exactamente (a, n) soluciones incongruentes mod n.
Demostracin: Del teorema 21, todas las soluciones de la ecuacin diofntica ax + ny = b
son de la forma x = x0 + nt/d, y = y0 at/d, d = (a, n), t Z, en donde x0 , y0 satisfacen
ax0 + ny = b. Dejando que t corra por los valores t = 0, 1, . . . ((a, n) 1), se obtiene
(a, n) soluciones que son mutuamente incongruentes, ya que el valor absoluto de la differencia de cualesquiera dos de ellas es menor que n. Si x = x0 + nt /d es cualquier otra
solucin, se escribe t como t = qd + r, 0 r < d. Entonces
x = x0 + n(qd + r)/d
= x0 + nq + nr/d
x0 + nr/d mod n.
Luego, toda solucin de la congruencia ax b mod n es congruente mod n a uno y
solamente uno de los d valores x0 + nt/d, 0 t d 1. Quirese decir que si existiese
una solucin de la congruencia, entonces existirn d soluciones incongruentes mod n.
49 Teorema Sean x, y enteros y sean a, n enteros diferentes de cero. Entonces

ax ay

mod n

si y slo si
xy
Demostracin:
implica que

mod

n
.
(a, n)

Si ax ay mod n entonces a(x y) = sn para algn entero s. Esto


(x y)

n
a
=s
.
(a, n)
(a, n)

Ya que (a/(a, n), n/(a, n)) = 1, se deduce que


n
|(x y),
(a, n)
gracias al lema de Euclides (lema 14). Esto implica que
xy

mod

n
.
(a, n)

20

Captulo 3
n
entonces se tendr
(a, n)
an
ax ay mod
,
(a, n)

Recprocamente, si se tiene x y mod

en multiplicando por a. Como (a, n) divide a a, la congruencia anterior implica, a fortiori


que ax ay = tn para algn entero t. Esto termina la demostracin.
50 Corolario Si ax ay mod n y (a, n) = 1, entonces x y mod n.

3.2.

Sistemas residuales completos y reducidos

51 Definicin Si a b mod n entonces b es llamado un residuo de a mdulo n. Un conjunto

{a1 , a2 , . . . an }
es llamado un sistema completo de residuos mdulo n si para cada entero b existe exactamente uno ndice
j para el cual b aj mod n. En particular, al conjunto {0, 1, . . . , n 1} se le llama conjunto cannico
de residuos mdulo n.
52 Definicin Sea n > 1. Los (n) enteros

1 = a1 < a2 < < a(n) = n 1


menores que n y relativamente primos a n reciben el nombre de residuos cannicos reducidos mdulo n.
53 Definicin Sea n > 1 un entero. Un entero b es llamado inverso multiplicativo de un entero a mdulo
n si ab 1 mod n.
54 Teorema Si existiese, el inverso multiplicativo x de un entero a mdulo el entero n > 1, es nico.

Demostracin: Si x, y fuesen inversos de a mod n entonces ax 1 mod n y tambin


ay 1 mod n. Multiplicando por y la primera congruencia, se ve que (ya)x y mod n.
As, x y mod n.
55 Teorema Sean n > 1, a enteros. Entonces a posee un inverso mdulo n si y slo si a es relativamente

primo a n.
Demostracin: Presmase que b es un inverso de a mod n. Luego ab 1 mod n que
conlleva la existencia de un entero s tal que ab 1 = sn, esto es, ab sn = 1. sta es una
combinacin lineal de a y n, luego ser divisible por (a, n), dando (a, n) = 1.
Recprocamente, si (a, n) = 1, por el teorema de Bachet-Bezout existe enteros x, y tales que
ax + ny = 1. Esto da de inmediato ax 1 mod n, lo cual quiere decir que a tiene como
inverso a x, mod n.

Sistemas residuales completos y reducidos

21

56 Corolario Si n > 1 es entero, entonces existe solamente (n) enteros invertibles, mdulo n.
57 Teorema Si a es relativamente primo al entero positivo integer n, entonces existe un entero positivo

k n tal que ak 1 mod n.


Demostracin: Ya que (a, n) = 1 se tiene (aj , n) = 1 para toda j 1. Considrese la
sucesin a, a2 , a3 , . . . , an+1 mod n. Como sta posee n + 1 trminos y hay slo n residuos
diferentes mdulo n, el principio de las pichoneras de Dirichlet implica que dos de estas
potencias estn en la misma clase residual mod n. sto es, se puede hallar s, t con 1 s <
t n+1 tal que as at mod n. Ahora bien, 1 ts n. Luego as at mod n resulta
en ats as ats at mod n, lo que quiere decir que at ats at mod n. Utilizando el
corolario 50 se deduce que ats 1 mod n, lo que demuestra el resultado.
Si (a, n) = 1, el teorema anterior dice que existe un entero positivo k con ak 1 mod n. Por el
axioma del buen orden, existe un entero mnimo satisfaciendo esta congruencia, lo que sugiere la prxima
definicin.
58 Definicin Si m es el entero positivo mnimo con am 1 mod n, entonces se side que a tiene orden

m mod n.

59 Teorema Sea n > 1 un entero. Entonces a Z tiene un orden mod n si y solamente si (a, n) = 1.

Demostracin: Si (a, n) = 1, entonces a tiene un orden en virtud del teorema 57 y el


axioma del buen orden. Presmase pues que a has an order mod n. Evidentemente a 6= 0.
La existencia de un orden conlleva la existencia de un entero positivo m tal que am 1
mod n. Luego, existe un entero s con am + sn = 1, o sea, a am1 + sn = 1. Esta es una
combinacin lineal de a y de n por lo cual es divisible por (a, n). De aqu se deduce que
(a, n) = 1.
60 Teorema Sea (a, n) = 1 y sea t un entero. Entonces at 1 mod n si y slo si ordn a|t.

Demostracin: Presmase que ordn a|t. Luego existe un entero s con sordn a = t, de donde
se deduce
at asordn a (aordn a )s 1s 1 mod n.
Recprocamente, presmase que at 1 mod n y que t = x ordn a + y, 0 y < ordn a.
Entonces
ay atxordn a at (aordn a )x 1 1x 1 mod n.
Si y > 0 entonces se tendra un entero positivo menor que ordn a poseyendo la propiedad
ay 1 mod n, lo que contradice la definicin de ordn a como el menor entero positivo con
esta propiedad. As pues y = 0 y entonces t = x ordn a, esto es, ordn a|t.

22

Captulo 3

61 Teorema Sea n > 1, a Z, (a, n) = 1. Si r1 , r2 , . . . , r(n) es un sistema de residuos reducidos mdulo

n, entonces tambin ar1 , ar2 , . . . , ar(n) es un sistema de residuos reducidos mdulo n.

Demostracin: Se necesita demostrar que los (n) enteros ar1 , ar2 , . . . , ar(n) son mutuamente incongruentes mod n. Supngase que ari arj mod n para algn i 6= j. Como
(a, n) = 1, se deduce del corolario 50 que ri rj mod n. sto contradice el hecho que las
rs son incongruentes, con lo que queda demostrado el teorema.
62 Corolario Sea n > 1, a, b Z, (a, n) = 1. Si r1 , r2 , . . . , r(n) es un sistema de residuos reducidos

mdulo n, entonces ar1 + b, ar2 + b, . . . , ar(n) + b es tambin un sistema de residuos reducido mdulo n.

3.3.

Teoremas de Fermat, Wilson y Euler

63 Teorema (Pequeo teorema de Fermat) Sea p un primo y a un entero tal que p 6 |a. Entonces

ap1 1

mod p.

Demostracin: Ya que (a, p) = 1, los enteros a 1, a 2, . . . , a (p 1) tambin forman un


sistema de residuos reducido mod p en vista del corolario 62. Luego pues
(a 1)(a 2) (a (p 1)) 1 2 (p 1)

mod p,

o sea,
ap1 (p 1)! (p 1)!

mod p.

Como ((p 1)!, p) = 1, se puede cancelar las (p 1)!s gracias al corolario 50.
64 Corolario Para todo primo p y para todo entero a,

ap a

mod p.

Demostracin: O bien p|a o p 6 |a. Si p|a, entonces a 0 ap mod p y no hay nada que demostrar. Si p 6 |a, el pequeo teorema de Fermat dice que p|ap1 1. As pues
p|a(ap1 1) = ap a, que da el resultado.
65 Corolario Sea p un primo y a un entero. Si p 6 |a entonces ordp a|p 1.

Demostracin: El resultado se consigue de inmediato por el teorema 60 y el pequeo teorema de Fermat.


66 Lema Si a2 1 mod p, o bien a 1 mod p o bien a 1 mod p.

Demostracin: Se tiene p|a2 1 = (a 1)(a + 1). Como p es primo, p debe dividir a al


menos uno de estos dos factores, lo que demuestra el lema.

Congruencias simultneas

23

67 Teorema (Teorema de Wilson) Si p es primo, entonces (p 1)! 1 mod p.

Demostracin: Si p = 2 o p = 3, el resultado se deduce por verificacin directa. Presmase


que p > 3. Considrese a, 2 a p 2. A cada a se le asocia su inverso nico a mod p,
i.e. aa 1 mod p. Observse que a 6= a porque si no, a2 1 mod p violando el lema
anterior, ya que a 6= 1, a 6= p 1. En nultiplicando las as con 2 a p 2, se aparean
stas con sus inversos, y la contribucin neta de este producto es por lo tanto 1. En smbolos,
2 3 (p 2) 1

mod p.

En otras palabras,

(p 1)! 1

2ap2

(p 1) 1 1 (p 1) 1

mod p.

Esto conlleva al resultado.


68 Teorema (Teorema de Euler) Si (a, n) = 1, entonces a(n) 1 mod n.

Demostracin: Sean a1 , a2 , . . . , a(n) los residuos cannicos reducidos mod n. Como


(a, n) = 1, aa1 , aa2 , . . . , aa(n) , tambin forman un sistema de residuos reducidos, gracias
al corolario 62. As pues,
aa1 aa2 aa(n) a1 a2 a(n)

mod n,

a(n) a1 a2 a(n) a1 a2 a(n)

mod n.

o
Como (a1 a2 a(n) , n) = 1, se puede cancelar el producto a1 a2 a(n) en uno y otro
lado, de donde se deduce el teorema.
69 Corolario Si (a, n) = 1, entonces ordn a|(n).

3.4.

Congruencias simultneas

70 Teorema (Teorema snico de los residuos o Teorema de Sun Tsu) Sean m1 , m2 , . . . mk enteros
relativamente primos por pares, todos mayores que 1, y sean a1 , a2 , . . . ak enteros arbitrarios. Luego el
sistema de congruencias
x a1
mod m1
x a2
mod m2
.. .. ..
. . .

x ak
posee una solucin nica mdulo m1 m2 mk .

mod mk

24

Captulo 3
m1 m2 mk
, 1 j k y sea Qj el inverso de Pj mod mj ,
mj
i.e., Pj Qj 1 mod mj , que sabemos que existe, ya que las mi son relativamente primas por
pares. Pngase
x = a1 P1 Q 1 + a2 P2 Q 2 + + a k Pk Q k .

Demostracin: Pngase Pj =

Este nmero claramente satisface las condiciones descritas en el teorema. La unicidad de esta
solucin es fcil de establecer mdulo m1 m2 mk .

3.5.

Criterios de divisibilidad

71 Teorema Un entero n es divisible por 5 si y solamente si su ltimo dgito es o un 0 o un 5.

Demostracin: Se derivar el resultado para n > 0, ya que para n < 0 slo basta aplicar el
teorema a n > 0. Como 10k 0 mod 5 para enteros k 1, se tiene
n = as 10s + as1 10s1 + + a1 10 + a0 a0

mod 5,

As pues, la divisibilidad de n por 5 depende en si a0 es divisible por 5, lo que slo pasa


cuando a0 = 0 o a0 = 5.
72 Teorema Sea k un entero positivo. Un entero n es divisible por 2k si y solamente si el nmero formado

por los ltimos k dgitos de n es divisible por 2k .


Demostracin: Si n = 0, no hay nada que demostrar. Si se demuestra el resultado para
n > 0 entonces el resultado para n < 0 se deduce al aplicar lo ya obtenido a n = (1)n > 0.
Presmase pues que n Z, n > 0 y sea su expansin decimal
n = as 10s + as1 10s1 + + a1 10 + a0 ,
donde 0 ai 9, as 6= 0. Ahora bien, 10t = 2t 5t 0 mod 2t para t k. Luego pues,
n = as 10s + as1 10s1 + + a1 10 + a0
ak1 10k1 + ak2 10k2 + + a1 10 + a0

mod 2k ,

de donde n es divisible por 2k si y solamente si el nmero formado por los ltimos k dgitos
de n es divisible por 2k .
73 Teorema (Regla de los 9s) Un entero n es divisible por 9 si y solamente si la suma de sus dgitos es

divisible por 9.
Demostracin: Si n = 0 no hay nada que demostrar. Si se demuestra el resultado para n > 0
entonces se puede deducir el resultado para n < 0 en aplicando lo ya obtenido al nmero
n = (1)n > 0. Presmase pues que n Z, n > 0 y que su expansin decimal es
n = as 10s + as1 10s1 + + a1 10 + a0 ,

Criterios de divisibilidad

25

donde 0 ai 9, as 6= 0. Observse que 10 1 mod 9 y que 10t 1t 1 mod 9. Ahora


bien,
n = as 10s + as1 10s1 + + a1 10 + a0
as + + a1 + a0 mod 9,

de donde se colige el resultado.

Como 10 1 mod 3 se puede tambin ver que n es divisible por 3 si y solamente si la


suma de su dgitos es divisible por 3.
74 Definicin Si el entero n tiene expansin decimal

n = as 10s + as1 10s1 + + a1 10 + a0 ,


entonces su suma alternante de dgitos es
as as1 + as2 as3 + + (1)s1 a0
75 Teorema Un entero n es divisible por 11 si y solamente si su suma de dgitos alternante es divisible

por 11.
Demostracin: Presmase que n > 0. Sea
n = as 10s + as1 10s1 + + a1 10 + a0 ,
donde 0 ai 9, as 6= 0. Observse que 10 1 mod 11y as 10t (1) mod 11.
Luego
n = as 10s + as1 10s1 + + a1 10 + a0
as (1)s + as1 (1)s1 + as2 (1)s2 + + a1 + a0
and se colige el resultado.

mod 11


Apendice

Ejemplos y problemas
A.1.

Sobre el primer captulo

76 Ejemplo Demustrese que

2 es irracional.

a
Resolucin: Presmase al contrario que 2 es racional, esto es, que hay enteros a, b con 2 = . Esto
b
implica que el conjunto

A = {n 2 : (n, n 2) (N \ {0})2 }
no esnulo ya que
contiene a a. Por el axioma del buen orden, A tiene un elemento mnimo, llmese
j = k 2. Como 2 1 > 0, se tiene que

j( 2 1) = j 2 k 2 = (j k) 2

es un entero positivo. Como 2 < 2 2 implica que 2 2 < 2 y tambin j 2 = 2k, se ve entonces que

(j k) 2 = k(2 2) < k( 2) = j.

As pues, (j k) 2 es un entero positivo de A menor que j. Esto contradice la eleccin de j como el


menor elemento de A y termina la demostracin. Contrstese este mtodo con el del ejemplo 114.
77 Problema Demustrese que no existe ningn entero en el intervalo ]0; 1[.
78 Ejemplo Sean a, b, c enteros a6 + 2b6 = 4c6 . Demustrese que a = b = c = 0.

Resolucin: Claramente basta considerar enteros positivos. Escjase un tro a, b, c que satisface la
ecuacin y con
max(a, b, c) > 0
tan pequeo como fuere posible. Si a6 +2b6 = 4c6 entonces a ha de ser par, dgase a = 2a1 . Esto conlleva
a 32a61 + b6 = 2c6 . Luego b es par, dgase b = 2b1 y por tanto 16a61 + 32b61 = c6 . De esto resulta que
c tambin es par, dgase c = 2c1 , y as a61 + 2b61 = 4c61 . Pero entonces max(a1 , b1 , c1 ) < max(a, b, c):
contradiccin. As todas las variables deben ser cero.
26

Sobre el primer captulo

27

79 Problema (IMO, 1988) Si a, b son enteros positivos para los cuales la cantidad

demustrese entonces

a 2 + b2
es un cuadrado perfecto.
1 + ab

a 2 + b2
es entera,
1 + ab

80 Ejemplo (USAMO, 1978) Llmese bueno al entero n si se pudiere escribir de la forma

n = a1 + a2 + + a k ,
en donde a1 , a2 , . . . , ak son enteros positivos, no necesariamente distintos y satisfaciendo
1
1
1
+
++
= 1.
a1 a2
ak
Dada la informacin de que todo entero desde el 33 hasta el 73 es bueno, demustrese que todo entero
33 es bueno.
Resolucin: Primero se demostrar que si n es bueno entonces tambin 2n + 8 y 2n + 9 son buenos.
Para esto presmase que n = a1 + a2 + + ak y que
1=

1
1
1
+
++
.
a1 a2
ak

Luego 2n + 8 = 2a1 + 2a2 + + 2ak + 4 + 4 y


1
1
1
1 1 1 1 1
+
++
+ + = + + = 1.
2a1 2a2
2ak 4 4 2 4 4
Adems, 2n + 9 = 2a1 + 2a2 + + 2ak + 3 + 6 y
1
1
1 1 1 1 1
1
+
++
+ + = + + = 1.
2a1 2a2
2ak 3 6 2 3 6
Luego si
si n es bueno entonces ambos 2n + 8 y 2n + 9 son buenos.

(A.1)

Sea P(n) la proposicin: todos los enteros n, n + 1, n + 2, . . . , 2n + 7 son buenos. Por hiptesis
P(33) es cierta. Pero (A.1) implica la veracidad de P(n + 1) cada vez que P(n) sea cierta. Luego la
asercin es demostrada por induccin fuerte.
81 Problema Demustrese, va induccin, que la expresin

33n+3 26n 27
es un mltiplo de 169 para todos los nmeros naturales n.
82 Ejemplo (Putnam, 1978) Sea A un conjunto cualquiera de 20 enteros tomados de la progresin arit-

mtica
1, 4, . . . , 100.
Comprubese que deber de haber dos enteros distintos en A cuya suma es 104.

28

Apndice A

Resolucin: Frmese una particin de los 34 elementos de la progresin en los 19 grupos


{1}, {52}, {4, 100}, {7, 97}, {10, 94} . . . {49, 55}.
Como se han de tomar 20 para formar el conjunto A, por el principio de las casillas de Dirichlet deben de
haber dos enteros que pertenezcan a uno de los pares, y por tanto suman a 104.
83 Problema Pngase en evidencia que entre siete enteros cualesquiera menores o iguales que 126, siem-

pre se podr hallar dos, llmense a y b, los cuales satisfacen las desigualdades
b < a 2b.
84 Problema Dados cualesquiera 10 enteros en el conjunto {1, 2, . . . , 99} demustrese que siempre habr

dos subconjuntos disjuntos cuyos elementos sumarn a la misma suma.


85 Problema No importa que 55 enteros se elija del conjunto

{1, 2, . . . , 100},
demustrese que siempre habr dos que difieren por 10.
86 Ejemplo Demustrese que el cuadrado de todo entero es de la forma 4k o de la forma 4k + 1. Luego

demustrese que ningn entero en la sucesin


11, 111, 1111, 11111, . . .
es el cuadrado de un entero.
Resolucin: Si el entero es par, es decir de la forma 2a, su cuadrado es (2a)2 = 4a2 , que es de la forma
4k. Si el entero es impar, digamos 2t + 1, entonces (2t + 1)2 = 4(t2 + t) + 1, que es de la forma 4k + 1.
Ahora bien, para n 2,
. . 11} 00 + 8 + 3 = 100 11
. . 11} +8 + 3 := 4s + 3,
11
. . . 1} = 11
| .{z
| .{z
| {z
n 1 s

n2 1 s

n2 1 s

en donde s = 25 11
. . 11} +2. As pues, todo nmero en esta sucesin es de la forma 4k + 3. Pero se
| .{z
n2 1 s

sabe que un cuadrado ha de tener la forma 4k 4k + 1 y por lo tanto ningn miembro de esta sucesin es
el cuadrado de un entero.

87 Problema (AHSME, 1976) Sea r el residuo cuando 1059, 1417 y 2312 se dividen por d > 1. Halle el

valor de d r.
88 Problema Demustrese que n2 + 23 es divisible por 24 para un nmero infinito de nmeros n.
89 Ejemplo Sea n > 0 entero.

Sobre el primer captulo

29

Sea a 6= 1. Demustrese la identidad


1 + a + a2 + + an1 =

an 1
.
a1

(A.2)

Demustrese la identidad
xn yn = (x y)(xn1 + xn2 y + xn3 y2 + + xyn2 + yn1 ).

(A.3)

De esto se deduce que si x, y son enteros con x 6= y entonces x y divide a xn yn .



Si n es impar, hgase patente que (x + y) (xn + yn ).


Demustrese que si k es un entero positivo impar
1k + 2k + + nk
es divisible por
1 + 2 + + n.
Resolucin: Para el primer inciso, se proceder por induccin. Para n = 1 es claro que 1 =
n = 2 es evidente que 1 + a + a2 =
para n + 1. Ahora bien

a1
, y para
a1

a3 1
. Suponiendo la validez de A.2 para n, habr de demostrarse
a1
an 1
+ an
a1
an 1 + an+1 an
=
a1
an+1 1
=
,
a1

(1 + a + a2 + + an1 ) + an =

demostrando la validez de A.2 para n + 1. As la primera asercin queda demostrada por induccin.


x
Para demostrar A.3 basta poner a = en A.2 y simplificar. Es evidente entonces que (x y) (xn
y
yn ).

Si n fuere impar, entonces (y)n = yn y con substituir y por y en A.3 se obtiene el resultado.
Para obtener la ltima aseveracin obsvese primero que
1+2++n =

n(n + 1)
.
2

Se considerar los casos cuando n es par y cuando n es impar por separado.


Presmase primero que n es par. Luego

n
2

es un entero y cada una de las expresiones

n2
1 + (n 1) ; 2 + (n 2) ; ;
2
k

n+2
+
2

n k k
;n ,
2

30

Apndice A

es divisible por

n
2

en vista del inciso anterior. Reagrupando de la manera

n k
n+2
1 + n ; 2 + (n 1) ; ;
+
2
2
k

tambin se ve que la suma es divisible por n + 1. Como


n(n + 1)
que la suma es divisible por
.
2
Presmase ahora que n es impar, de donde

n+1
2

n
2

n1
1 + n ; 2 + (n 1) ; ;
2
k

y n + 1 no tienen factores en comn, se deduce

es un entero. Cada una de las expresiones

n+3
+
2

n+1
;
2

es divisible por n+1


2 en vista del inciso anterior. De igual manera la suma es divisible por n ya que cada
una de las expresiones

n1
1 + (n 1) ; 2 + (n 2) ; ;
2
k

lo es. Como

n+1
2

n+1
+
2

; nk ,

y n no tienen factores en comn, se deduce que la suma es divisible por

n(n+1)
.
2

90 Problema Demustrese que el entero

11
. . 11}
| .{z
221 1 s

es compuesto.

91 Problema Demustrese que 100 divide a 1110 1.


92 Problema Demustrese que 7 divide a 22225555 + 55552222 .
93 Problema Demustrese que si 2n 1 es un nmero primo, entonces n es un nmero primo. Primos

de esta forma se llaman primos de Mersenne.


94 Problema Demustrese que si 2n + 1 es un nmero primo, entonces n es una potencia de 2. Primos

de esta forma se llaman primos de Fermat.


95 Problema ((UM)2 C4 , 1987) Dado que 1002004008016032 tiene un factor primo p > 250000, en-

cuntrese.
96 Ejemplo Demustrese que el nico primo de la forma n4 + 4 es el 5.

Resolucin: Se puede restringir el argumento a enteros positivos. Obsrvese que


n4 + 4 = n4 + 4n2 + 4 4n2
= (n2 + 2)2 (2n)2
= (n2 2n + 2)(n2 + 2n + 2).

Sobre el primer captulo

31

Si este producto es un nmero primo entonces el factor ms pequeo debe ser igual a 1. As n2 2n + 2 =
1, o sea (n 1)2 = 0, esto es n = 1. As, el nico primo de esta forma es 14 + 4 = 5.
97 Problema Demustrese que

a3 + b3 + c3 3abc = (a + b + c)(a2 + b2 + c2 ab bc ca).


Luego demustrese que

6 (a + b + c) = 6 (a3 + b3 + c3 ).

Demustrse adems que que si n es la suma de tres cubos consecutivos, entonces 9 n.


98 Problema Comprubese que el producto de cuatro enteros consecutivos, diferentes de 0, jams es un

cuadrado.
99 Problema Demustrese que si k es impar entonces 2n+2 divide a
n

k2 1
para todos los enteros naturales n 1.
100 Problema Demustrese que entre tres enteros siempre se pueden escoger dos tales que a3 b ab3

sea divisible por 10.


101 Ejemplo Demustrese que el producto de n enteros consecutivos es divisible por n!.

Resolucin: Obsrvese que el problema se reduce a considerar enteros estrictamente positivos, ya que si
fuesen estrictamente negativos, con multiplicar por (1)n no se afecta la divisibilidad, y si incluyesen al
0, el producto sera 0, que es definitivamente divisible por n!.
Premase pues que todos los enteros en consideracin son estrictamente positivos. Se utilizar el axioma del buen orden (Axioma 2) y se argir por contradiccin. Si M fuese el menor entero para el cual
(M + 1)(M + 2) (M + n)
n!
no es entero. Obsrvese que M > 0 ya que

n!
= 1 es entero. Ahora bien,
n!

(M + 1) (M + n) = (M + 1) (M + n 1)(M + n)
= M(M + 1)(M + 2) (M + n 1)
+(M + 2) (M + n 1)n.
Por definicin de M,

n! (M(M + 1)(M + 2) (M + n 1))

32

Apndice A

(n 1)! ((M + 2) (M + n 1)).

Luego n! ((M + 2) (M + n 1)n). Pero entonces





n! ((M + 1)(M + 2) (M + n)),


lo cual es una contradiccin.

102 Problema Demustrese que 6 (n3 n) y que 120 (n5 5n3 + 4n) para todo entero n.
103 Problema La suma de enteros positivos es 1996. Cul es el valor mximo de su producto?
104 Problema Hallse todos los enteros positivos de la forma

1
r+ ,
r
donde r es un nmero racional.
105 Ejemplo Demustrese que si n es un entero positivo tal que 2n + 1 es un cuadrado, entonces n + 1

es la suma de dos cuadrados consecutivos.


Resolucin: Como 2n + 1 es un cuadrado impar, tenemos 2n + 1 = (2t + 1)2 para algn entero t.
Resolviendo para n,
(2t + 1)2 1
= 2t2 + 2t.
n=
2
Luego n + 1 = t2 + (t + 1)2 , la suma de dos cuadrados consecutivos.
106 Problema Demustrese que si 3n + 1 es un cuadrado, entonces n + 1 es la suma de tres cuadrados.
107 Problema Demustrese que si n > 11 entonces n se puede escribir como la suma de dos nmeros

compuestos.
108 Ejemplo Sean m, n, a 6= 1 enteros positivos. Demustrese que

MCD (am 1, an 1) = aMCD(m,n) 1.


Resolucin: Pngase d = MCD (m, n), sd = m, td = n. Entonces am 1 = (ad )s 1 es divisible por

ad 1 y de manera semejante, an 1 es divisible by ad 1. As (ad 1) MCD (am 1, an 1).


Ahora bien, en virtud al Teorema de Bachet-Bezout (Teorema 12) existen enteros x, y con mx + ny =
d. Ntese que x, y habrn de tener signos opuestos (no pueden ser ambos negativos, ya que d sera
entonces negativo. Si ambos fuesen positivos entonces d m + n, lo que contradice al hecho que d

Sobre el primer captulo

33

m, d n). Presmase pues,


sin prdida de generalidad,
que x > 0, y 0. Pngase t = (am 1, an 1).


Entonces t (amx 1) y t (any 1). Luego t ((amx 1) ad (any 1)) = ad 1, estableciendo


el resultado.
109 Problema (IMO, 1959) Comprubese que la fraccin

n.

21n + 4
es irreducible para todo entero natural
14n + 3

110 Problema (AIME, 1985) Los nmeros de la sucesin

101, 104, 109, 116, . . .


son de la forma an = 100 + n2 , n = 1, 2, . . .. Para cada n pngase dn = MCD (an , an+1 ). Hllese
max dn .
n1

os,
1899) Comprubese que para todo entero positivo n, la expresin
111 Problema (Eotv

2903n 803n 464n + 261n


es siempre divisible por 1897.
112 Problema Tmese cualesquiera 51 enteros de entre 1, 2, . . . , 100. Demustrese que hay al menos

dos que son relativamente primos.


113 Problema Demustrese que todo entero n > 6 puede ser escrito como la suma de dos enteros ambos
mayores que 1 tales que cada sumando sea relativamente primo.
114 Ejemplo Demustrese que

2 es irracional.

Resolucin: Supngase a miras de contradiccin que 2 = a/b con enteros positivos a, b relativamente
primos. Entonces 2b2 = a2 . El lado siniestro de esta ecuacin tiene un nmero impar de factores primos
mientras que el diestro tiene un nmero par. Esto contradice el Teorema fundamental de la aritmtica
(Teorema 15). Contrstese este mtodo con el del ejemplo 76.
115 Problema Demustrese que si el polinomio

p(x) = a0 xn + a1 xn1 + + an1 x + an


con coeficientes ntegros alcanza el valor de 7 para cuatro valores ntegros de x entonces no puede tomar
el valor de 14 para ningn valor ntegro de la variable x.
116 Problema Demustrese que el producto de tres enteros estrictamente positivos consecutivos jams

ser una potencia perfecta.


117 Problema Comprubese que m5 + 3m4 n 5m3 n2 15m2 n3 + 4mn4 + 12n5 nunca ser igual

a 33.

34

Apndice A

118 Problema Demustrese que la suma

S = 1/2 + 1/3 + 1/4 + + 1/n


jams es ntegra.
119 Problema Demustrese que existe un entero nico n para el cual 28 + 211 + 2n es un cuadrado

perfecto.

120 Ejemplo Si MCD (a, b) = 1 entonces MCD a + b, a2 ab + b2 = 1 3.

Resolucin: Sea d = MCD a + b, a2 ab + b2 . Ahora bien, d divide a


(a + b)2 a2 + ab b2 = 3ab.



As, d divide a 3b(a + b) 3ab = 3b . De manera semejante se deduce que d 3a2 . Pero entonces
2

d MCD 3a2 , 3b2 = 3 MCD a2 , b2 = 3 MCD (a, b)2 = 3.


121 Ejemplo Demustrese que dados cualesquiera 33 enteros diferentes con todos sus factores primos en

el conjunto {5, 7, 11, 13, 23}, siempre hay dos distintos cuyo producto es un cuadrado.
Resolucin: Cada uno de los 33 enteros es de la forma
5a 7b 11c 13d 23f .
As, a cada uno de los 33 enteros se les puede asociar un vector de la forma (a, b, c, d, f). Dependiendo la
paridad de los componentes del vector, hay 32 = 25 tipos de estos vectors. Por ejemplo, uno de los tipos es
(par, par, impar, impar, par). Pinsese de estas 32 clases de vectores como en 32 casillas. A los 33 enteros
se les distribuir en las 32 casillas y por tanto, una de las casillas tendr al menos dos enteros diferentes.
En esta casilla los exponentes de los nmeros son de la misma paridad, as que al multiplicarse, todos los
exponentes sern pares. Luego este producto ser un cuadrado.
122 Problema (IMO 1985) Dado un conjunto M de 1985 enteros positivos distintos, ninguno de cuyos

factores primos es mayor que 26, demustrese que M siempre tendr cuatro elementos distintos cuyo
producto es una cuarta potencia.
123 Problema Tmese 51 enteros cualesquiera del conjunto

{1, 2, . . . , 100}.
Demustrese que siempre habr dos, uno dividiendo al otro.
124 Problema Demustrese que si ambos p es primo, o bien 8p 1 es primo y 8p + 1 compuesto o

viceversa.

Sobre el segundo captulo

35

125 Ejemplo Hllese MCD (23, 29) mediante el algoritmo de Euclides y Encuntrese soluciones enteras

para la ecuacin 23x + 29y = 1.


Resolucin: Se tiene
29 = 1 23 + 6,
23 = 3 6 + 5,
6 = 1 5 + 1,
5 = 5 1.
El ltimo residuo no nulo es el 1 de donde MCD (23, 29) = 1.
Ahora bien,
1 = 6 1 5,
5 = 23 3 6,
6 = 29 1 23.
Luego,
1 =
=
=
=
=

615
6 1 (23 3 6)
4 6 1 23
4(29 1 23) 1 23
4 29 5 23.

Esto resuelve la ecuacin con x = 5, y = 4.

126 Problema Encuntrese un nmero infinito de soluciones para

23x + 29y = 1.
127 Problema Hllese soluciones enteras para la ecuacin

23x + 29y = 7.

A.2.

Sobre el segundo captulo

128 Problema Demustrese que si n es compuesto entonces (n) n

n. Cuando se verificar la

igualdad?

129 Problema (AIME, 1992) Encuntrese la suma de todos los nmeros racionales positivos que sean

menores que 10 y tengan denominador 30 cuando se escriban en trminos mnimos..


130 Problema Demustrese que (n) n2(n) .
131 Problema Demustrese que (n) >

n para n > 6.

36

Apndice A



132 Problema Demustrese que si (n) n, entonces n debe de ser de la forma 2a 3b para enteros no

negativos a, b.

133 Problema Demustrese que si (n) n 1, entonces n no es divisible por ningn cuadrado mayor

que 1.
134 Problema (Mandelbrot 1994) Cuatrocientas personas se colocan alrededor de un crculo. Se marca

a una persona, se perdona a las prximas k personas, luego se marca a otra, se perdona a las prximas
k, etctera, continuando hasta que se marque a una persona por segunda vez. Para cuntos valores de k
menores que 400 se marcarn a todas las personas en el crculo al menos una vez?



135 Problema Demustrese que si (n) n1 y n es compuesto, entonces n tiene al menos tres factores

primos distintos.



136 Problema Demustrese que si (n) n 1 y n es compuesto, entonces n tiene al menos cuatro

factores primos distintos.


137 Ejemplo (El problema de los casilleros) La guardarropa de un gimnasio tiene 100 casilleros y

100 usuarios. Al principio, todos los casilleros estn abiertos. Entra el usuario nmero 1 y cierra todos
los casilleros. Entra el usuario nmero 2, y cierra todos los casilleros con nmero par. Entra el usuario
nmero 3 y cambia de estado (de cerrado a abierto o vice versa) todos los casilleros cuyo nmero es un
mltiplo de 3. Entra el usuario nmero 4 y cambia de estado (de cerrado a abierto o vice versa) todos los
casilleros cuyo nmero es un mltiplo de 4. Sucede as sucesivamente hasta que entra el usuario nmero
100 y cambia de estado el casillero nmero 100. Qu casilleros permanecen cerrados?
Resolucin: Se ver que los casilleros cuyo nmero es un cuadrado perfecto permanecen cerrados. As
los casilleros nmero 1, 4, 9, 16, 25, 36, 49, 64, 81 y 100 son los que permanecen cerrados. Obsrvese
que el casillero nmero n es afectado por el usuario d si y slo si d divide a n. As slo aquellas n que
tengan un nmero impar de divisores permaneceran cerradas. Ahora bien, cada factor d de n se puede
n
n
aparear con , y as, n tendr un nmero impar de factores si y slo si se tiene d = , esto es, si n es un
d
d
cuadrado perfecto.
138 Problema Descrbase todos los enteros positivos n para los cuales d(n) = 10.
139 Problema Demustrese que

d(2n 1) d(n).
140 Problema Demustrese que d(n)

3n verificndose la igualdad si y slo si n = 12.

Sobre el segundo captulo

37

141 Problema Demustrese que se cumple la expansin

d(n)tn =

X
n=1

n=1

tn
,
1 tn

llamada expansin de Lambert.


142 Problema Pngase d1 (n) = d(n), dk (n) = d(dk1 (n)), k = 2, 3, . . .. Descrbase dk (n) para k
lo suficientemente grande.
143 Problema Dado m N, demustrese que el conjunto



A = {n N : m d(n)}
posee una progresin aritmtica infinita.
144 Problema Sea n un nmero perfecto. Demustrese que

X1


d n

145 Problema Demustrese que

Y


d n

= 2.

d = nd(n)/2 .

146 Problema (AIME 1995) Sea n = 231 319 . Cuntos divisores positivos de n2 son menores que n pero

no dividen a n?
147 Problema Demustrese que si n es compuesto, entonces (n) > n +

n.

148 Problema Demustrese que la ecuacin (n) = n + k, donde k > 1 es un nmero natural fijo, posee

un nmero finito de soluciones.


149 Problema Caractercese todos los enteros positivos n para los cuales (n) es impar.
150 Problema Demustrese que p es primo si y solamente si (p) = 1 + p.
151 Problema Demustrese que

1
1
(n!)
1+ ++ .
n!
2
n

38

Apndice A

152 Problema Demustrese que ninguna potencia de un primo no puede ser un nmero perfecto. Luego,

demustrese que de existir un nmero perfecto impar, ste debe tener al menos dos factores primos distintos.
153 Problema Demustrese que si n es un nmero perfecto impar, entonces solamente uno de sus fac-

tores primos ocurre con potencia par.


154 Problema Demustrese que

n
X
k=1

(k) =

n
X
j=1

n
jT U.
j

155 Problema Hllese todos los conjuntos de enteros positivos {a, b, c} tales que a b c = 462.
156 Ejemplo Hllese un polinomio no idnticamente nulo P(x, y) tal que para todo nmero real t se
tenga P(T2tU, T3tU) = 0.

Resolucin: Comprobarase que 3T2tU 2T3tU = 0, 1 o 2. Luego entonces se podr elegir


P(x, y) = (3x 2y)(3x 2y 1)(3x 2y + 1)(3x 2y + 2).
Para verificar la asercin, obsrvese que como TxU tiene perodo unitario, es suficiente comprobar la
asercin para t [0; 1[. Divdase [0; 1[ as
[0; 1[= [0; 1/3[[1/3; 1/2[[1/2; 2/3[[2/3; 1[.
Si t [0; 1/3[, entonces tanto T2tU como T3tU son = 0, y as 3T2tU 2T3tU = 0. Si t [1/3; 1/2[
entonces T3tU = 1 y T2tU = 0, y as 3T2tU 2T3tU = 2. Si t [1/2; 2/3[, luego T2tU = 1, T3tU = 1,
dando 3T2tU 2T3tU = 1. Si t [2/3; 1[, se tendr T2tU = 1, T3tU = 2, y 3T2tU 2T3tU = 1.

157 Ejemplo Descrbase todos los enteros positivos n para los cuales 1 + T 2nU 2n.

Resolucin:
Sea 2n = m(1 + T 2nU). Si m T 2nU
1 entonces 2n (T 2nU
1)(T 2nU +

2nU + 1, entonces 2n (T 2nU2 + 1)2


1) = T 2nU2 1 2n 1 < 2n, contradiccin. Si m T
2n + 1, otra contradiccin. Por lo tanto se debe tener m = T 2nU.

l(l + 1)
Recprocamente, sea n =
. Ya que l < 2n < l + 1, se tiene l = T 2nU. Luego todos los
2
enteros con la propiedad deseada son nmeros triangulares.
158 Ejemplo Demustrese que la sucesin de enteros

n
T 1+ 2 U

donde n es un entero no negativo, es alternadamente par e impar.

Sobre el segundo captulo

39

Resolucin: En virtud del teorema del binomio,

(1 + 2) + (1 2) = 2

0kn/2

(2)

n
:= 2N,
2k

n
n

2
<
0,
entonces
(1

2)
es
la
parte
fraccionaria
de
(1
+
2) o de
un nmero
par.
Como
1
<
1

n
(1 + 2) + 1 dependiendo de la paridad de n. Para n impar,

(1 + 2)n 1 < (1 + 2)n + (1 2)n < (1 + 2)n ,


n
n
n
n
de donde
(1
+
2)
+
(1

2)
=
T(1
+
2)
U,
que
es
siempre
par,
y
para
n
par
2N
:=
(1
+
2) +
n
n
n
(1 2) = T(1 + 2) U + 1, y as T(1 + 2) U = 2N 1, es siempre impar para n par.
159 Ejemplo Demustrese que los primeros mil dgitos de

(6 + 35)1980
luego del punto decimal son todos 9s.
Resolucin: Gracias al ejemplo 158,

(6 + 35)1980 + (6 35)1980 = 2k,

1
< 6 35, y al cuadrar 3500 < 3481, cones un entero par. Ahora bien, 0 < 6 35 < 1/10 (si no
10

tradiccin) y luego 0 < (6 35)1980 < 101980 . Se deduce que


2k 1 + 0,9
. . . 9} = 2k
| {z
1979 nines

1
101980

< (6 + 35)1980 < 2k,

demostrando la asercin.

160 Ejemplo (Putnam 1948) Si n es un entero positivo, demustrese que

T n + n + 1U = T 4n + 2U.
Resolucin: Elevando al cuadrado se ve que

4n + 1 < n + n + 1 < 4n + 3.
Ni 4n + 2 ni 4n + 3 son cuadrados, ya que todo cuadrado es congruente a 0 o 1 mdulo 4, luego

T 4n + 2U = T 4n + 3U,
de donde se colige el resultado.
161 Ejemplo Hllese una frmula para el ensimo entero positivo no cuadrado.

40

Apndice A

Resolucin: Sea Tn el ensimo entero positivo no cuadrado. Luego, existe un nmero entero positivo m
tal que m2 < Tn < (m + 1)2 . Ya que hay m cuadrados menores que Tn se tiene que Tn = n + m. Entonces
se ve que
m2 < n + m < (m + 1)2
o sea
m2 m < n < m2 + m + 1.
Como n, m2 m, m2 + m + 1 son todos enteros, las desigualdades anteriores implican que
m2 m +

1
1
< n < m2 + m + ,
4
4

1
esto es, (m 1/2)2 < n < (m + 1/2)2 . Pero entonces m = T n + U. Luego el ensimo entero positivo
2

no cuadrado es Tn = n + T n + 1/2U.

162 Ejemplo (Putnam 1983) Sea f(n) = n + T nU. Demustrese que para cada entero positivo m, la

sucesin
m, f(m), f(f(m)), f(f(f(m))), . . .
tiene al menos el cuadrado de un entero.
Resolucin: Sea m = k2 + j, 0 j 2k. Divdase las ms en dos grupos: aqullas (grupo A) para las
que j, 0 j k y stas (grupo B) para las que j, k < j < 2k + 1.
2
2
Obsrvese que k2 m < (k
+ 1) = k + 2k + 1. Si j 2= 0, no hay nada que2 demostrar. Supngase
primero que m B. Como T mU = k, se tiene f(m) = k + j + k = (k + 1) + j k 1, con 0
j k 1 k 1 < k + 1. Esto quiere decir que f(m) o bien es un
cuadrado, o bien f(m) A. As pues,
es slo necesario considerar la alternativa m A, en cuyo case T m + kU = k y
f(f(m)) = f(m + k) = m + 2k = (k + 1)2 + j 1.
Esto significa que o bien f(f(m)) es un cuadrado, o bien f(f(m)) A con un exceso j 1 menor que
el exceso j de m. Luego de cada iteracin el exceso se reduce, y eventualmente ser cero, en cuyo caso se
obtendr un cuadrado.
163 Ejemplo Resulvase la ecuacin

Tx2 x 2U = TxU,
para x R.
Resolucin:
Obsrvese que TaU = TbU si y slo si k Z with a, b [k, k + 1), lo que sucede si y slo


si a b < 1. Luego, la ecuacin dada tendr solucin si y solamente si x2 2x 2 < 1, de donde el


conjunto solucin es

1
1
1
x R : x ] 1 : (1 5)] [ (1 + 17), (1 + 21)[ .
2
2
2

Sobre el segundo captulo

41

164 Ejemplo Encuntrese la parte entera


6

10
X
1
.
k
k=1

Resolucin: La funcin x 7 x1/2 es decreciente. Luego, para todo entero positivo k,


1

<
k+1

Z k+1
k

dx
1
< .
x
k

Sumando de k = 1 hasta k = 10 1 se deduce que


Z 106
106
106 1
X
1
dx X 1
<
<
.
x
k 1
k
k=2
k=1
Se verifica fcilmente que la integral es 1998. Luego
6

10
X
1
3
< 1999.
1998 + 1/10 <
k
k=1

La parte entera es as 1998.


165 Ejemplo En cuntos ceros termina 300!?

Resolucin: El nmero de ceros queda determinado por la potencia mayor de 10 que divida a 300!. Ya
que abundan ms los mltiplos de 2 en 300! que los mltiplos de 5, el nmero de ceros queda determinado
por la potencia mayor de 5 que divida a 300!. En virtud de la frmula de De Polignac, la buscada potencia
es

X
T300/5k U = 60 + 12 + 2 = 74.
k=1

1000
166 Ejemplo Divide 7 a
?
500
Resolucin: La potencia mayor de 7 que divide a 1000! is T1000/7U + T1000/72 U + T1000/73 U =
142 + 20 + 2 = 164, gracias a la frmula de De Polignac (teorema 39).
De manera semejante, la potencia
!
1000!
1000
mayor de 7 que divide a 500! is 71 + 10 + 1 = 82. Ya que
=
, la potencia mayor de 7
500
(500!)2
!
!
1000
1000
es 164 2 82 = 0, de donde se colige que el 7 no divide a
.
que divide a
500
500
167 Ejemplo Sea n = n1 + n2 + + nk donde los ni son enteros no negativos. Demustrese que la

cantidad

es entera.

n!
n1 !n2 ! nk !

42

Apndice A

Resolucin: Por 3 del teorema 38 se deduce mediante induccin que


Ta1 U + Ta2 U + + Tal U Ta1 + a2 + + al U.

Por la frmula de De Polignac (teorema 39) la potencia de un primo p que divide a n!


X
X
Tn/pj U =
T(n1 + n2 + + nk )/pj U.
j1

j1

Luego la potencia de p que divide a n1 !n2 ! nk ! es


X
Tn1 /pj U + Tn2 /pj U + Tnk /pj U.
j1

Ya que
Tn1 /pj U + Tn2 /pj U + + + Tnk /pj U T(n1 + n2 + + nk )/pj U,

se colige que la potencia de cualquier primo que divida al numerador de


n!
n1 !n2 ! nk !
es al menos tan grande como la potencia del mismo primo que divida al denominador. Esto demuestra la
asercin.
168 Ejemplo Dado un entero n > 3, demustrese que el mnimo comn mltiplo de los productos, x1 x2 xk (k
1), cuyos factores xi son enteros positivos satisfaciendo

es menor que n!.

x1 + x2 + xk n,

Resolucin: Se demostrar que el mnimo comn mltiplo en cuestin es


Y
pTn/pU .
p

p primo

Considrese un producto arbitrario x1 x2 xk , y un primo arbitrario p. Supngase que pj divide a xj


pero que pj +1 no divide a xj . Claramente, p1 + + pk n y como p p, se tiene
n
p(1 + k ) n o 1 + + k T U.
p
Luego, se sigue que el exponente de un primo arbitrario dividiendo el mnimo comn mltiplo p es a lo
sumo Tp/nU. Pero en tomando x1 = = xk = p, k = Tn/pU, se observa que para al menos un producto
se consigue la igualdad. Esto demuestra la asercin.
169 Ejemplo Supngase que se criba los enteros positivos de la manera siguiente: se toma a1 = 1 y se
deja a1 + 1 = 2. El prximo trmino es 3, al que se llamar a2 y tomar, y luego se dejar a2 + 2 = 5.
El prximo entero disponible es 4 = a3 , y luego se dejar a3 + 3 = 7, etc. As se tomar los enteros
1, 3, 4, 6, 8, 9, 11, 12, 14, 16, 17, . . . . Encuntrese una frmula para an .

Resolucin: Se pide una sucesin {Sn } complementaria a {Sn + n}. Por el teorema de Beatty (teorema
42), TnU
y TnU + n = Tn( + 1)U son complementarias si 1/ + 1/( + 1) = 1. Pero luego =

(1 + 5)/2, la razn dorada. El ensimo trmino es pues an = TnU.

Sobre el tercer captulo

A.3.

43

Sobre el tercer captulo

170 Ejemplo Demustrese que 7 divide a 22225555 + 55552222 utilizando congruencias. Esta pregunta

se vi ya en el problema 92.
Resolucin: 2222 3 mod 7, 5555 4 mod 7 y 35 5 mod 7. Ahora bien, 22225555 + 55552222
35555 + 42222 (35 )1111 + (42 )1111 51111 51111 0 mod 7, lo que demuestra la asercin.
171 Ejemplo Hallse el residuo cuando 61987 es dividido por 37.

Resolucin: 62 1 mod 37. As pues, 61987 661986 6(62 )993 6(1)993 6 31 mod 37.
172 Ejemplo (USAMO, 1979) Determnese todas las soluciones no negativas

(n1 , n2 , . . . , n14 )
de la ecuacin diofntica
n41 + n42 + + n414 = 1599
de haberlas.
Resolucin: No hay tales soluciones. Todas las cuartas potencias mod 16 son o bien 0 o bien 1
mod 16. Esto significa que
n41 + + n414
es a lo sumo 14 mod 16. Pero 1599 15 mod 16.
173 Problema Hallse el ltimo dgito de 3100 .
7

174 Problema Hallse el dgito de las unidades de 77 .


175 Problema Comprubese que 7 divide a 32n+1 + 2n+2 para todo nmero entero n > 0.
176 Problema (Olimpada polaca) Qu dgitos debe substituirse por a y b en 30a0b03 de tal manera

que el entero resultante sea divisible por 13 ?


177 Problema Comprubese que la ecuacin x2 5y2 = 2 no tiene soluciones enteras.
178 Problema Comprubese la siguiente observacin de Euler: 232 + 1 es divisible por 641.
179 Problema Hallse un nmero infinito de enteros n tal que 2n + 27 sea divisible por 7.
180 Problema Existe acaso enteros positivos x, y tal que x3 = 2y + 15?

44

Apndice A

181 Problema Comprubese que 2k 5, k = 0, 1, 2, . . . nunca deja residuo 1 cuando es dividido por 7.
182 Ejemplo Encuntrese todas las soluciones de 5x 3 mod 7

Resolucin: De acuerdo al teorema 48 existe una solucin nica a la congruencia mod 7 por ser (5, 7) =
1. Gracias al algoritmo de Euclides,
7 = 51+2
5 = 22+1
2 = 2 1.
As,
1 = 522
2 = 7 5 1,
dando
1 = 5 2 2 = 5 2(7 5 1) = 5 3 7 2.
De aqu, 3 = 5(9) 7(6). De sto resulta que 5 9 3 mod 7, o sea, 5 2 3 mod 7. As pues x 2
mod 7.
183 Ejemplo Resulvase la congruencia

3x 6

mod 12.

Resolucin: Como (3, 12) = 3 y 3|6, la congruencia tiene tres soluciones incogruentes. Por inspeccin
x = 2 es na solucin. En virtud del teorema 21, todas las soluciones son de la forma x = 2 + 4t, t Z.
Poniendo t = 0, 1, 2, se obtienen las tres soluciones incongruentes t = 2, 6, 10 mdulo 12.
184 Ejemplo Encuntrese el inverso de 5 mod 7.

Resolucin: Bscase una solucin a 5x 1 mod 7. Por inspeccin se ve que la solucin buscada es
x 3 mod 7.
185 Ejemplo Hllese el orden de 8 mod 11.

Resolucin: Por el corolario ?? ord11 8 es uno de entre 1, 2, 5 o 10. Ahora 82 2 mod 11, 84 4
mod 11 and 85 1 mod 11. El orden es pues ord11 8 = 10.
186 Ejemplo Sea a1 = 4, an = 4an1 , n > 1. Hllese el residuo cuando a100 se divide por 7.

Resolucin: Por el pequeo teorema de Fermat, 46 1 mod 7. Como 4n 4 mod 6 para todos los
enteros positivos n, se tiene 4n = 4 + 6t para algn entero t. As
a100 4a99 44+6t 44 (46 )t 4

mod 7.

187 Ejemplo Demustrese que m, n Z, mn(m60 n60 ) es siempre divisible por 56786730.

Sobre el tercer captulo

45

Resolucin: Obsrvese que a = 56786730 = 2 3 5 7 11 13 31 61. Sea Q(x, y) = xy(x60 y60 ).


Obsrvese que (x y)|Q(x, y), (x2 y2 )|Q(x, y), (x3 y3 )|Q(x, y), (x4 y4 )|Q(x, y), (x6
y6 )|Q(x, y), (x10 y10 )|Q(x, y), (x12 y12 )|Q(x, y), y (x30 y30 )|Q(x, y).
Si p es cualquiera de los primos dividiendo a a, se tiene entonces que mp m 0 mod p y que
np n 0 mod p. As n(mp m) m(np n) 0 mod p, sto es, mn(mp1 np1 ) 0
mod p. Luego se tiene que
2|mn(m n)|Q(m, n), 3|mn(m2 n2 )|Q(m, n), 5|mn(m4 n4 )|Q(m, n),
7|mn(m6 n6 )|Q(m, n), 11|mn(m10 n10 )|Q(m, n),
13|mn(m12 n12 )|Q(m, n), 31|mn(m30 n30 )|Q(m, n)
y 61|mn(m60 n60 )|Q(m, n). Como todos estos son primos distintos, se deduce que a|mnQ(m, n),
como se quera demostrar.
188 Ejemplo Si p 1 mod 4, demustrese que

p1
! 1
2

mod p.

Resolucin: En el producto (p1)!, se aparea j, 1 j (p1)/2 con pj. Obsrvese que j(pj)
j2 mod p. Luego
1 (p 1)!

1j(p1)/2

j (1)

(p1)/2

p1
!
2

mod p.

El resultado se consigue al observar que (1)(p1)/2 = 1.


189 Ejemplo (IMO 1970) Demustrese que para todo entero n es imposible partir el conjunto

{n, n + 1, n + 2, n + 3, n + 4, n + 5}
en dos subconjuntos tales que el producto de los miembros del uno sea igual al producto de los miembros
del otro.
Resolucin: Supngase en miras de contradiccin que existiese tal n con tal particin, el primer subconjunto teniendo producto de miembros A y el otro teniendo producto de miembros B. O bien, uno de los
enteros en {n, n + 1, n + 2, n + 3, n + 4, n + 5} es divisible por 7, en cuyo caso exactamente uno de
entre A o B es divisible por 7, y por lo tanto, A B no es divisible por 72 y as A B no es un cuadrado. En
la segunda posibilidad, todos los miembros del conjunto son relativamente primos a 7. Esto quiere decir
que, gracias al teorema de Wilson,
n(n + 1) (n + 6) 1 2 6 A B 1

mod 7.

Pero si A = B entonces A2 1 mod 7, lo que es imposible ya que 1 no es cuadrado mod 7.

46

Apndice A

190 Problema Sea n N, n 2. Demustrese que si N est la suma de n nmeros impares consecutivos,

entonces N no puede ser primo.

191 Ejemplo (Putnam, 1956) Demustrese que todo entero positivo posee un mltiplo cuya expansin

decimal involucra todos los 10 dgitos.


Resolucin: Se n un entero positivo arbitrario con k dgitos, y sea m = 123456780 10k+1 . Entonces
los n consecutivos m + 1, m + 2, . . . m + n comienzan en 1234567890, forman un sistema completo
de residuos mdulo n y luego uno de ellos es divisible por n.
192 Ejemplo Para cuntos enteros n en {1, 2, 3, . . . , 100} es el dgito de las decenas de n2 impar?

Resolucin: En el subconjunto {1, 2, . . . 10} hay slo dos valores de n (4 y 6) para los cuales el dgito de
las decenas de n2 es impar. Ahora bien, (n + 10)2 = n2 + 20n + 100 tiene la misma paridad en su dgito
de las decenas que el dgito de las decenas de n2 . Luego, hay 2 10 = 20 enteros n para los cuales se
verifica la condicin prescrita.
193 Ejemplo Hallar todos los enteros con dgito inicial 6 tales que si se les suprime este dgito incial,el

nmero resultante es 1/25 del nmero original.


Resolucin: Sea x el entero buscado. Entonces x = 610n +y donde y es un entero positivo. La condicin
del problema estipula que
1
(6 10n + y) ,
y=
25
o sea,
10n
y=
= 25 10n2 .
4
Esto requiere n 2 y por lo tanto y = 25, 250, 2500, 25000, etc.. Luego x = 625, 6250, 62500, 625000,
etc..
194 Ejemplo Sea A un entero positivo y A sea el entero positivo resultante de alguna permutacin

especfica de los dgitos de A. Demustrese que si A + A = 1010 entonces A es divisible por 10.
Resolucin: Claramente, A y A debern tener 10 dgitos cada uno. Pongas pues
A = a10 a9 a8 . . . a1
y
A = b10 b9 b8 . . . b1 ,

donde ak , bk , k = 1, 2, . . . , 10 son los dgitos de A y A respectivamente. Ahora, como A+A = 10000000000,


deberemos tener que a1 + b1 = a2 + b2 = = ai + bi = 0 y
ai+1 + bi+1 = 10, ai+2 + bi+2 = = a10 + b10 = 9,
para algn subndice i, 0 i 9. Note que si i = 9 no hay ninguna suma de las ai+2 + bi+2 , ai+3 +
bi+3 , . . . y si i = 0 no hay ninguna suma de las a1 + b1 , . . . , ai + bi .

Sobre el tercer captulo

47

Sumando,
a1 + b1 + a2 + b2 + + ai + bi + ai+1 + bi+1 + + a10 + b10 = 10 + 9(9 i).
Ahora bien, si i es par, 10 + 9(9 i) es impar y si i es impar 10 + 9(9 i) es par. Pero como
a1 + a2 + + a10 = b1 + b2 + + b10 ,
tenemos
a1 + b1 + a2 + b2 + + ai + bi + ai+1 + bi+1 + + a10 + b10 = 2(a1 + a2 + + a10 ),

un entero par. Colegimos que i es impar, lo que necesariamente implica a1 = b1 = 0, esto es, A y A son
ambos divisibles por 10.
195 Ejemplo Demustrese que todos los enteros en la sucesin

49, 4489, 444889, 44448889, 44


. . 44} 88
. . 88} 9
| .{z
| .{z
n 4 s

son cuadrados.

n1 8 s

Resolucin: Obsrvese que


. . 88} 9 = 44
. . 44} 10n + 88
. . 88} 10 + 9
44
. . 44} 88
| .{z
| .{z
| .{z
| .{z
n 4 s

n1 8 s

n 4 s

=
=
=

n1 8 s

8
4
(10n 1) 10n + (10n1 1) 10 + 9
9
9
4
4
1
2n
n
10 + 10 +
9
9
9
1
2
n
(2 10 + 1)
9

2 10n + 1 2
3

Falta ahora demostrar que esta ltima cantidad es entera, esto es, que 3 divide a 2 10n + 1 = 2 00
. . 00} 1.
| .{z
n1 0 s

Pero la suma de los dgitos de esta ltima cantidad es 3, y por lo tanto este entero es divisible por 3.

196 Ejemplo Puede encontrarse un milln de enteros positivos que sean divisibles por al menos un

cuadrad?
Resolucin: La respuesta es afirmativa. Sean p1 , p2 , . . . , p1000000 un milln de primos diferentes. Por el
teorema snico de los residuos existe una solucin del siguiente sistema de congruencias:
x
x
..
.

..
.

1
2
..
.

x 1000000

..
.

mod p21 ,
mod p22 ,
mod p21000000 .

Los nmeros x + 1, x + 2, . . . , x + 1000000 son un milln de enteros consecutivos, cada uno divisible
por el cuadrado de un primo.


Apendice

Indicaciones y respuestas
77 Presmase a la contraria que el conjunto S de enteros en ]0; 1[ es no nulo. Siendo un conjunto de
enteros positivos, gracias al axioma del buen orden, este debe tener un elemento mnimo al que se llamar
m. Ahora bien, 0 < m2 < m < 1, y por lo tanto m2 S . Pero esto declara que S tiene un entero positivo,
m2 , que es menor que su elemento mnimo! Esta contradiccin establece que S = .
a 2 + b2
= k fuere un contraejemplo de un entero que no es un cuadrado perfecto con
1 + ab
max(a, b) tan pequeo como fuere posible. Puede presumirse, sin perder generalidad, que a < b, ya que
si a = b entonces
2a2
< 2,
0<k= 2
a +1
lo que fuerza k = 1, un cuadrado perfecto.

79 Supngase que

Ahora bien, a2 + b2 k(ab + 1) = 0 es una ecuacin cuadrtica en b cuya suma de races es ka y


cuyo producto de races es a2 k. Si b1 , b son sus races se tiene que b1 + b = ka y b1 b = a2 k.
Como a, k son enteros positivos, el suponer b1 < 0 es incompatible con a2 + b21 = k(ab1 + 1). Como
k se supone no ser un cuadrado perfecto, el suponer b1 = 0 es incompatible con a2 + 02 = k(0 a + 1).
Adems
a 2 k b2 k
b1 =
<
< b.
b
b
a2 + b21
As pues, se ha encontrado otro entero positivo, b1 para el cual
= k y el cual es menor que
1 + ab1
max(a, b): contradiccin. Por lo tanto k debe ser un cuadrado perfecto.
81 Para n = 0 se declara que 33 27 == 169 0 es un mltiplo de 169, lo cual es evidente. Presmase
que la asercin es cierta para n 1, n > 1, esto es, presmase que
33n 26n 1 = 169N
para algn entero N. Entonces
33n+3 26n 27 = 27 33n 26n 27 = 27(33n 26n 1) + 676n
48

Indicaciones y respuestas

49

lo cual se reduce a
27 169N + 169 4n,
que es evidentemente un mltiplo de 169. Queda demostrada la asercin por induccin.
83 Dividnse los nmeros {1, 2, 3, . . . , 126} en los seis conjuntos
{1, 2}, {3, 4, 5, 6}, {7, 8, . . . , 13, 14}, {15, 16, . . . , 29, 30},
{31, 32, . . . , 61, 62} y {63, 64, . . . , 126}.
Por el principio de las casillas de Dirichlet, dos de los siete nmeros yacern en el mismo conjunto, donde
obviamente, satisfacen las desigualdades estipuladas.
84 Hay 210 1 = 1023 subconjuntos no nulos posibles en un conjunto de 10 elementos. A cada uno
de estos conjuntos no vacos asciese la suma de estos 10 elementos. El valor mximo que estas sumas
pueden tener es 90 + 91 + + 99 = 945 < 1023. Luego, deben de haber al menos dos subconjuntos,
llmense A y B con la misma suma, ya que hay ms subconjuntos que sumas. Si los subconjuntos tuviesen
una interseccin no nula, basta considerar A \ (A B) y B \ (A B), que tambin tienen la propiedad
deseada.
85 Obsrvese primeramente que al elegir n + 1 enteros de cualquier conjunto de 2n enteros consecutivos,
siempre habr dos que diferirn por n. En efecto, al parear los 2n enteros consecutivos
{a + 1, a + 2, a + 3, . . . , a + 2n}
en los n pares
{a + 1, a + n + 1}, {a + 2, a + n + 2}, . . . , {a + n, a + 2n},
se ve que al elegir and n + 1 siempre habr dos que pertenecen al mismo grupo.
Agrupse pues los 100 enteros como sigue:
{1, 2, . . . 20}, {21, 22, . . . , 40},
{41, 42, . . . , 60}, {61, 62, . . . , 80}
y
{81, 82, . . . , 100}.
Si se eligieren 55 siempre habr once proviniendo del mismo grupo. Y en ese grupo en particular siempre
habr dos difiriendo por 10.
87 Por el algoritmo de divisin, hay enteros q1 , q2 , q3 con 1059 = dq1 + r, 1417 = dq2 + r y 2312 =
dq3 + r. Restando se obtiene 1253 = d(q3 q1 ), 895 = d(q3 q2 ) y 358 = d(q2 q1 ). Como
7179, 895 = 5179, 358 = 2179, se ve que d = 179. Como 1059 = 5179+164, r = 164. Finalmente,
d r = 15.
88 Se tiene que n2 + 23 = n2 1 + 24 = (n 1)(n + 1) + 24. Luego, las familias n = 24m 1, m =
0, 1, 2, 3, . . . producen infinitos valores de n2 + 23 que son divisibles por 24.

50

Apndice B

10a 1 10b 1
de
la
forma

, para ciertos enteros a, b.


90 Exprsese 11
.
.
.
11
| {z }
10b 1 10 1

221 1 s

91 1110 1 = (115 1)a = (11 1)(114 + 113 + 112 + 11 + 1)a.


92

22225555 + 55552222 = (22225555 + 45555 ) + (55552222 42222 ) (45555 42222 ).


Para otro punto de vista, vase el ejemplo 170.
93 Pngase n = pm, donde p es primo y m > 1. Utilcese la identidad A.3 para factorizar (2p )m 1.
k

94 Pngase n = 2k m, donde m > 1 es impar. Factorice (22 )m + 1.


95 Si a = 103 , b = 2 entonces
1002004008016032 = a5 + a4 b + a3 b2 + a2 b3 + ab4 + b5 =

a 6 b6
.
ab

Esta ltima expresin factoriza como


a 6 b6
= (a + b)(a2 + ab + b2 )(a2 ab + b2 )
ab
= 1002 1002004 998004
= 4 4 1002 250501 k,
en donde k < 250000. Por lo tanto p = 250501.
98 Sean n 1, n, n + 1, n + 2 cuatro enteros consecutivos. Entonces su producto P es
P = (n 1)n(n + 1)(n + 2) = (n3 n)(n + 2) = n4 + 2n3 n2 2n.
Ahora bien,
(n2 + n 1)2 = n4 + 2n3 n2 2n + 1 = P + 1 > P.
Como P 6= 0 y P es 1 ms que un cuadrado, P no puede ser un cuadrado.
99 La asercin es evidente para n = 1, ya que k2 1 = (k 1)(k + 1) es producto de dos nmeros
pares consecutivos, y por tanto uno es divisible por 2 y el otro por 4 as que el producto es divisible por 8.

Presmase que 2n+2 k2 1. Como k2

n+1

1 = (k2 1)(k2 + 1), se puede notar que 2n+2 divide



a (k2n 1), as que el problema se reduce a demostrar que 2 (k2n + 1). Pero esto es obvio, ya que como
k2n es impar k2n + 1 es par.
100 Obsrvese que a3 b ab3 = ab(a b)(a + b) es siempre par, no importa que enteros sean substituidos. Si uno de los enteros es de la forma 5k, entonces no hay ms que demostrar. Si no, entonces se
escogen tres enteros de entre enteros de la forma 5k 1 (casilla I) 5k 2 (casilla II). Por el principio
de las casillas de Dirichlet, dos de estos tres enteros debern caer en la misma casilla as que hay dos que
o bien su suma o bien su diferencia es divisible por 5, lo que establece el resultado.

Indicaciones y respuestas

51

102 Obsrvese que n3 n = (n1)n(n+1) y que n5 5n3 +4n = (n2)(n1)n(n+1)(n+2)


y utilcese el ejemplo 101.
103 Se consideran enteros positivos a1 , a2 , . . . , an con a1 + a2 + + an = 1996. Es claro que para
maximizar a1 a2 an , ninguna de las ak s puede ser igual a 1. Se demostrar que para obtener un
producto mximo se deber tener la mayora de las ak = 3 y a lo sumo dos aj = 2. Supongse que aj > 4.
Al substituir aj por los dos trminos aj 3 y 3 la suma no se afecta, pero el producto incrementa pues
aj < 3(aj 3). As pues las ak s son iguales a 2, 3 4. Pero como 2 + 2 + 2 = 3 + 3 y 2 2 2 < 3 3,
si hay tres o ms 2s, se pueden substituir con 3s. Como 1996 = 3(665) + 1 = 3(664) + 4, el producto
mximo es pues 3664 4.
104 Se demostrar que la expresin r + 1/r es entera slo cuando r = 1, en cuyo caso r + 1/r = 2. Sea
pues
1
r + = k,
r
k un entero positivo. Luego

k k2 4
.
r=
2
Como k es un entero, r puede ser entero si y slo si k2 4 es un cuadrado de la misma paridad que k.
Ahora, si k 3,
(k 1)2 < k2 4 < k2 ,
esto es, k2 4 est entre dos cuadrados consecutivos y por lo tanto no puede ser un cuadrado. Si k = 1,
k2 4 no es real. Si k = 2, k2 4 = 0. Luego, r + 1/r = 2, esto es, r = 1. Esto termina la demostracin.

106 Es claro que 3n + 1 no es un mltiplo de 3, luego 3n + 1 = (3k 1)2 . De aqu

(3k 1)2 1
+ 1 = 3k2 2k + 1 = k2 + k2 + (k 1)2 ,
3
como queramos demostrar.
n+1 =

107 Si n es par, escrbase n = n 6 + 6. Como n > 11, n 6 es par y mayor que 2, por tanto compuesto.
Si n es impar, n = n 9 + 9. Como n > 11, n 9 es par y mayor que 2, por lo tanto compuesto.
109 Ntese que 2(21n + 4) 3(14n + 3) = 1, de donde el mximo comn divisor divide a 1. As
pues, el numerador y el denominador no pueden compartir a un factor mayor que 1.
110 Se tiene que

dn = MCD 100 + n2 , 100 + (n + 1)2

2
= MCD 100 + n2 , 100 + n
+
2n
+
1

= MCD 100 + n2 , 2n + 1 .
As pues

d (2(100 + n2 ) n(2n + 1)) = 200 n.



Por lo tanto, dn (2(200n)+(2n+1)) = 401, de donde se colige que dn 401 para toda n. Alcanzar
dn un valor tan grande como 401? Efectivamente! Para n = 200 se tiene que a200 = 100 + 2002 =
100(401) y que a201 = 100 + 2012 = 40501 = 101(401). Luego se deduce que max dn = 401.
n1

52

Apndice B

111 Gracias a A.3, 2903n 803n es divisible por 2903 803 = 2100 = 7 300 y 261n 464n es
divisible por 203 = (29) 7. Por lo tanto, la expresin es divisible por 7. Adems 2903n 464n es
divisible por 2903 464 = 2439 = 9 271 y 803n + 261n es divisible por 803 + 261 = 542 =
2 271. As pues, como la expresin es divisible por 7 y por 271 y como estos son relativamente primos,
la expresin es pues divisible por 7 271 = 1897.
112 Pngase los 100 enteros en los 50 pares
{1, 2}, {3, 4}, {5, 6} . . . , {99, 100}.
Como se elegir 51 enteros, deber de haber dos de entre ellos que yazgan en el mismo par. stos son
relativamente primos.
113 Si n es impar escrbase a = 2, b = n2, n = a+b y como n2 es impar se tiene MCD (n 2, 2) =
1. Si n es par, entonces o bien n = 4k o bien n = 4k + 2. Si n = 4k, escrbase a = 2k + 1, b = 2k 1,
n = a+b, los cuales son relativamente primos al ser dos enteros impares consecutivos. Si n = 4k+2, k > 1
escrbase a = 2k + 3, b = 2k 1, n = a + b, los que de la misma manera son relativamente primos (si
d = MCD (2k + 3, 2k 1) entonces d divide a 2k + 3 (2k 1) = 4. Luego d es 1, 2 4 pero como
d es impar es por fuerza = 1).
115 Ntese primeramente que 7 puede descomponerse en a lo sumo tres factores diferentes: 7 = 7(1)(1).
Si p(ak ) 7 = 0 para cuatro valores distintos ak , 1 k 4 entonces
p(x) 7 = (x a1 )(x a2 )(x a3 )(x a4 )q(x)
para algn polinomio q con coeficientes enteros. Presmase ahora que existe un entero m con p(m) = 14.
Entonces
7 = p(m) 7 = (m a1 )(m a2 )(m a3 )(m a4 )q(m).
Como los factores m ak , la igualdad anterior descompone a 7 en cuatro o ms valores distintos, lo que
es una contradiccin.
116 Sea el producto (n1)n(n+1) = (n2 1)n, n > 1. Como n2 1 y n son relativamente primos, por
el Teorema fundamental de la aritmtica (Teorema 15) n2 1 es una potencia k-sima perfecta (k 2)
y tambin lo es n. Esto implica que tanto n2 1 y n2 son potencias k-simas perfectas consecutivas, lo
que es imposible. Obsrvese que ni n2 1 ni n2 son ni 0 ni 1.
117 Obsrvese que
m5 + 3m4 n 5m3 n2 15m2 n3 + 4mn4 + 12n5
= (m 2n)(m n)(m + n)(m + 2n)(m + 3n),
un producto de cinco factores. Ahora bien, 33 se puede descomponer a lo sumo en cuatro factores distintos:
33 = (11)(3)(1)(1). Si n 6= 0, todos los factores de arriba son distintos y no pueden dar 33 en virtud
del Teorema fundamental de la aritmtica (Teorema 15) porque un producto de cinco factores diferentes
no puede igualar a un producto de cuatro factores diferentes. Si n = 0, el producto de los factores es m5
pero 33 no es una quinta potencia.

Indicaciones y respuestas

53

118 Sea k el mximo entero que satisface 2k n y sea P el producto de todos los enteros impares menores
1
o iguales a n. El nmero 2k1 PS es una suma cuyos trminos, excepto el 2k1 PS k , son enteros.
2
119 Si k2 = 28 + 211 + 2n = 2304 + 2n = 482 + 2n , entonces k2 482 = (k 48)(k + 48) = 2n .
Gracias a la propiedad de factorizacin nica, k 48 = 2s , k + 48 = 2t , s + t = n. Luego 2t 2s = 96 =
3 25 o 2s (2ts 1) = 3 25 . Por factorizacin nica, s = 5, t s = 2, dando s + t = n = 12.
122 Los nmeros de M son de la forma
2a 3b 5c 7d 11f 13g 17h 19j 23k .
Los diez exponentes dan 512 = 210 vectores de paridad. Luego entre cualesquiera 513 elementos de M
siempre se hallar dos cuyo producto es un cuadrado.
Se poda ahora los cuadrados. Como 1985 > 513 se puede hallar un par de nmeros distintos a1 , b1
tales que a1 b1 = c21 . Qutese este par, dejando 1983 enteros. De estos 1983 enteros, se puede hallar un
par a2 , b2 tales que a2 b2 = c22 . Remuvase este par, dejando 1981 enteros. De estos 1981 enteros, se
puede encontrar un par a3 , b3 tales que a3 b3 = c23 . Esta operacin de remover se puede continuar n + 1
veces, en donde n es el mayor entero positivo que satisface 1985 2n 513, esto es, n = 736. Luego
se puede recoger 737 pares ak , bk tales que ak bk = ck sea un cuadrado. Como 737 > 513, se puede
encontrar un par de las cm tales que su producto ci cj sea un cuadrado. Pero como cada una de las cm es
un cuadrado a su vez, el producto ci cj = a2 ser una cuarta potencia.
123 Cualquier entero puede escribirse de la manera 2a m, en donde m es impar. Tan slo hay 50 enteros
impares en el conjunto dado, luego hay slo m posibilidades para m. Luego dos de los 51 enteros elegidos

deben de ser de la forma 2a m y otro de la forma 2a m. Luego el menor de estos dos dividir al mayor.
124 Si p = 3, entonces 8p 1 = 23 y 8p + 1 = 25, luego la aseveracin se cumple para p = 3. Si p > 3, p
es de la forma 3k+1 o de la forma 3k+2. Si p = 3k+1 entonces 8p1 = 24k7 y 8p+1 = 24k6,
que es divisible por 6 y por lo tanto no es primo. Si p = 3k + 2 entonces 8p 1 = 24k 15 no es primo
al ser divisible por 3.
126 Gracias al ejemplo 125 se tiene que el par x0 = 5, y0 = 4 es una solucin. Se puede encontrar una
familia infinita de soluciones en poniendo
x = 5 + 29t, y = 4 23t, t Z.
127 Del ejemplo 125 se tiene 23(5) + 29(4) = 1. Multiplicando uno y otro lado por 7,
23(35) + 29(28) = 7,
lo que resuelve el problema.
129 400
173 Quirese hallar 3100 mod 10. Obsrvese que 32 1 mod 10. Luego, 3100 = (32 )50 (1)50
1 mod 10. As, el ltimo dgito es el 1.

54

Apndice B
7

174 Se tiene que hallar 77 mod 10. Ahora bien, como 72 1 mod 10, entonces tenemos 73 72
7 7 3 mod 10 y 74 (72 )2 1 mod 10. Adems, 72 1 mod 4 y por lo tanto 77 (72 )3 7
3 mod 4, lo que quiere decir que hay un entero t tal que 77 = 3 + 4t. Ensamblando todo esto,
7

77 74t+3 (74 )t 73 1t 3 3

mod 10.

As el ltimo dgito es un 3.
175 Obsrvese que 32n+1 3 9n 3 2n mod 7 y 2n+2 4 2n mod 7. Luego
32n+1 + 2n+2 7 2n 0

mod 7,

para todo nmero natural n.


176 Como 30a0b03 = 3 + 100b + 10000a + 3000000, observamos que 30a0b03 3 + 9b + 3a +
3 6 + 9b + 3a mod 13. Para que 30a0b03 sea divisible por 13 necesitamos 9b + 3a 7 mod 13.
Aqu claro est, se tendr 0 a, b 9. Por inspeccin se ve que a = 8, b = 1;a = 5, b = 2; a = 2, b = 3;
a = 9, b = 5; a = 6, b = 6; a = 3, b = 7; a = 0, b = 8. Luego 3080103, 3050203, 3020303, 3090503,
3060603, 3030703, 3000803 son todos divisibles por 13.
177 Si x2 = 2 5y2 , entonces x2 2 mod 5. Pero 2 no es un cuadrado mod 5.
178 Obsrvese que 641 = 27 5 + 1 = 24 + 54 . Luego 27 5 1 mod 641 y 54 24 mod 641.
Ahora bien, 27 5 1 mod 641 nos da 54 228 = (5 27 )4 (1)4 1 mod 641. Esta tima con

gruencia y 54 24 mod 641 nos da 24 228 1 mod 641, lo que significa que 641 (232 + 1).

179 Obsrvese que 21 2, 22 4, 23 1, 24 2, 25 4, 26 1 mod 7 y as 23k 1 mod 3 para


todos los enteros positivos k. Luego 23k + 27 1 + 27 0 mod 7 para todos los enteros positivos k.
Esto produce una familia infinita de enteros n = 3k, k = 1, 2, . . . tal que 2n + 27 es divisible por 7.
180 No. Los cubos mod 7 son 0, 1, y 6. Ahora bien, cada potencia de de 2 es congruente con 1, 2, 4
mod 7. As pues, 2y + 15 2, 3, 5 mod 7. Esto es imposible.
181 21 2, 22 4, 23 1 mod 7 y este ciclo de tres se repite. As pues, 2k 5 deja residuos 3, 4, 6
al ser dividido por 7.

Вам также может понравиться